You are on page 1of 55

P

R E

-F O

U N D A T IO N

B Y

MANISH

MATHEMATICS

KUMAR

TRIGONOMETRY

INTRODUCTION
Trigonometry is the branch of Mathematics which deals with the measurement of angles and sides of a
triangle.
The word Trigonometry is derived from three Greek roots : trio meaning thrice or Three,gonia meaning an
angle and metron meaning measure. Infact, Trigonometry is the study of relationship between the sides and
the angles of a triangle.
Trigonometry has its application in astronomy, geography, surveying, engineering and navigation etc. In the past,
astronomers used it to find out the distance of stars and plants from the earth. Even now, the advanced technology
used in Engineering are based on trigonometric concepts.
In this chapter, we will define trigonometric ratios of angles in terms of ratios of sides of a right triangle. We will
also define trigonometric ratios of angles of 00, 300, 450, 600, and 900. We shall also establish some identities
involving these ratios.

HISTORICAL FACTS
Indian Mathematician have established keen interest in the study of Trigonometry since ages. They are known for
their innovation in the use of size instead the use of choid. The most outstanding astronomer has been Aryabhatta.
Aryabhatta was born in 476 A.D. in Kerala. He studied in the university
of Nalanda. In mathematics, Aryabhattas contribution are very valuable.
He was the first mathematician to prepare tables of sines. His book
Aryabhatta deals with Geometry, Mensuration, Progressions, Square
root, Cube root and Celestial sphere (spherical Trigonometry). This
work, has won him recognition all over the world because of its logical
and unambiguous presentation of astronomical observations.
Aryabhatta was the pioneer to find the correct value of the constant

Circumference
up to four decimals as
with respect to a circle
Diameter

3.1416. he found the approximate value of and indirectly suggested


that is an irrational number. His observations and conclusions are very
useful and relevant today.
Greek Mathematician Ptolemy, Father of Trigonometry proved the
equation sin2A + cos2A = 1 using geometry involving a relationship
between the chords of a circle. But ancient Indian used simple algebra to
calculate sin A and cos A and proved this relation. Brahmagupta was the
first to use algebra in trigonometry. Bhaskarcharaya II (1114 A.D.) was
very brilliant and most popular Mathematician. His work known as
Siddhantasiron mani is divided into four parts, one of which is
Goladhyayas spherical trigonometry.

R E

-F O

U N D A T IO N

B Y

MANISH

KUMAR

MATHEMATICS

BASE, PERPENDICULAR AND HYPOTENUSE OF A RIGHT TRIANGLE


In ABC, if B = 900, then :
(i)
For A, we have :
(ii)

Base = AB, Perpendicular = BC and Hypotenuse = AC.


For C, we have :
Base = BC, Perpendicular = AB and Hypotenuse = AC.

So, in a right angled triangle, for a given angle,


(i)
The side opposite to the right angle is called hypotenuse.
(ii)
The side opposite to the right angle is called perpendicular.
(iii)
The third side (r.e., the side forming the given angle with the hypotenuse), is called base.

TRIGONOMETRICAL RARIOS (T-RATIOS) OF AN ANGLE


In ABC, let B = 900 and let A be acute.
For A, we have :
Base = AB, Perpendicular = BC and Hypotenuse = AC.
The T-ratios for A are defined as :
(i)
(ii)
(iii)
(iv)
(v)
(vi)

Perpendicular ( P) BC

, written as sin A.
Hypotenuse ( H )
AC
Base ( B)
AB

Cosine A =
, written as cos A.
Hypotenuse ( H ) AC
Perpendicular ( P) BC

Tangent A =
, written as tan A.
Base ( B)
AB
Hypotenuse ( H )
AC

, written as cosec A.
Cosecant A =
Perpendicular ( P) BC
Hypotenuse ( H ) AC

, written as sec A.
Secant A =
Base ( B)
AB
Base ( B)
AB

, written as cot A.
Cotangent A =
Perpendicular ( P) BC
Sine A =

Thus, there are six trigonometrical ratios based on the three sides of a right angled triangle.
Aid to Memory : The sine, cosine, and tangent ratios in a right triangle can be remembered by representing
them as strings of letters, as in SOH-CAH-TOA.
Sine = Opposite Hypotenuse
Cosine = Adjacent Hypotenuse
Tangent = Opposite Adjacent
The memorization of this mnemonic can be aided by expanding it into a phrase, such as Some Officers
Have Curly Auburn Hair Till Old Age.

R E

-F O

U N D A T IO N

B Y

MANISH

MATHEMATICS

KUMAR

RECIPROCAL RELATIONS
Clearly, we have :
(i) cos ec

1
sin

(ii) sec

1
cos

(iii) cot

1
tan

Thus, we have :
(i) sin cosec = 1

(ii) cos sec = 1

(iii) tan cot = 1

QUOTAENT RELATIONS

Consider a right angled triangle in which for an acute angle , we have :

Perpendicular P
Base
B

: cos =

Hypotenuse
H
Hypotenuse H
P
sin H
P H P
Now,

= tan (by def.)


B H B B
cos
H
B
cos H
B H B
and,

= cos (by def.)


P H P P
sin
H
sin
cos
Thus, tan =
and cot =
cos
sin
sin =

POWER OF T-RATIOS

We denote :
(i) (sin )2 by sin2 ;
and so on.
REMARK :

(i)

(ii)

(iii)
Ex.1
Sol.

(ii) (cos )2 by cos2 ; (iii) (sin )3 by sin3 ; (iv) (cos )3 by sin3 ;


The symbol sin A is used as an abbreviation for the sine of the angle A. Sin A is not
the product of sin and A. sin separated from A has no meaning. Similarly, cos A is
not the product of cos and A. similar interpretations follow for other trigonometric
ratios also.
We may write sin2 A, cos2 A, etc., in place of (sin A)2, (cos A)2, etc., respectively. But
cosec A = (sin A)-1 sin-1 A (it is called sine inverse A). sin-1 A has a different
meaning, which will be discussed in higher classes. Similar conventions hold for the
other trigonometric ratios as well.
Since the hypotenuse is the longest side in a right triangle, the value of sin A or cos A is
always less than 1 (or, in particular, equal to 1).

Using the information given in fig. write the values of all trigonometric ratios of angle C.
Using the definition of t-ratios,

AB 8 4
;
AC 10 5
BC 6 3
cotC =
;
AB 8 4

SinC =

BC 6 3

AC 10 5
AC 10 5
tanC =
and
BC 6 3

cosC =

AB 6 4
;
BC 8 3
AC 10 5
cosC =

AB 8 4
tanC =

R E

-F O

U N D A T IO N

B Y

MANISH

MATHEMATICS

KUMAR

3
. find the remaining trigonometric ratios of A.
4

Ex.2

In a right ABC, if A is acute and tan A =

Sol.

Consider a ABC in which B = 900


For A, we have :
Base = AB, Perpendicular = BC and Hypotenuse = AC.

Perpendicular 3

Base
4
BC 3

AB 4

tan A =

Let, BC = 3x units and AB = 4x units.


Then,

AC =

AB 2 BC 2

4 x 2 3x 2

25x 2

= 5x units.

Ex.3

In a ABC, right angled at B, if tan A =

Sol.

(i) sinA cosC + cosA sinC


We know that
tanA =

(ii) cosA cosC sinA sinC.

[NCERT]

BC
1

AB
3

BC : AB = 1 :

Let BC = k and AB =

Then,

1
, find the value of
3

AC =
=

3
3k
AB 2 BC 2 (Phythagoras theorem)

( 3k ) 2 (k ) 2 3k 2 k 2

4k 2 = 2k
BC k 1
sinA =

AC 2k 2
=

Now,

cosA =

AB

AC
BC

cosC =
AC
sinC =

and

AB
3k
3

AC
2k
2
3k
3

2k
2
BC k 1

AC 2k 2

(i)

sinA cosC + cosA sinC =

(ii)

cosA cosC sinA sinC =

1 1
3 3 1 3
,
.
1
2 2 2 2 4 4
31 1 3
3
3
.

0
2 2 2 2
4
4
4

R E

-F O

U N D A T IO N

B Y

MANISH

MATHEMATICS

KUMAR

1
2 tan A
, verify that 2 sinA cosA =
2
1 tan 2 A

Ex.4

If sinA =

Sol.

We know that
SinA =

BC 1

AC 2

Let

BC = k and AC = 2k

AB =
=

Now
and
Now

AC 2 AB 2

(2k ) 2 k 2 4k 2 k 2 3k 2 3k

AB

AC
BC
cosA =

AB
cosA =

Sol.

1 3
3

(i)
2 2
2
1
2
2
3 3 3 2 3 3
2
1
3
3 4 2
1
1

3
4
3

2 sinA cosA = 2. .

2 tan A

and
1 tan 2 A
Ex.5

3k
3

2k
2
k
1

3k
3

(ii)

In PQR, right angled at Q, PR + QR = 25 cm and PQ = 5 cm. Find the value of sinP, cosP and tanP.
[NCERT]
We are given
PR + QR = 25 cm

PR = (25 QR) cm
By Pythagoras theorem,
PR2 = QR2 + PQ2
or
(25 QR)2 = QR2 + 52
or
625 + QR2 50 QR = QR2 + 25
or
50QR = 625 25 = 600

QR = 12 cm.
and PR = (25 12) cm = 13 cm
Now

and

QR 12

PR 13
PQ 5
cosP =

PR 13
QR 12
tanP =

PQ 5
sinP =

R E

-F O

U N D A T IO N

B Y

MANISH

MATHEMATICS

KUMAR

Ex.6
Sol.

If A and Q are acute angles such that sin B = sinQ, then prove that B = Q.
Consider two right ABC and PQR such that sin B = sinQ.
We have,
sinB =

[NCERT]

AC
PR
and, sinQ =
PQ
AB

sinB = sinQ

AC PR

AB PQ

AC AB
k, (say)

PR PQ

(i)

AC = k PR and AB = k PQ
(ii)
Using Pythagoras theorem in triangles ABC and PQR, we have
AB2 = AC2 + BC2 and PQ2 = PR2 + QR2

BC =

AB 2 AC 2 and QR =

PQ 2 PR 2

BC

QR

AB 2 AC 2

k 2 PQ 2 k 2 PR 2

2
2
BC k PQ PR

=k
QR
PQ 2 PR 2

PQ 2 PR 2

BC

QR

PQ 2 PR 2

[Using (ii)]

From (i) and (ii), we have,

TRIGONOMETRICAL RATIO OF STANDARD ANGLES


T-Ratios of 450
Consider a ABC in which B = 900 and A = 450
Then, clearly, C = 450.

AB = BC = a (say).
AC =

AB 2 CB 2 a 2 a 2 2a 2 2a.

1
BC
a

:
AC
2a
2
1
AB
a
cos 450 =

:
AC
2a
2
BC a
tan 450 =
1
AC a
1
1
1
cosec 450 =
2 ; sec 450 =
2 ; cot 450 =
1
0
0
sin 45
cos 45
tan 450
sin 450 =

R E

-F O

U N D A T IO N

B Y

MANISH

MATHEMATICS

KUMAR

T-Ratios of 600 and 300


Draw an equilateral ABC with each side = 2a.
Then, A = B = C = 600.
From A, draw AD BC.
Then, BD = DC = a, BAD = 300 and ADB = 900.
Also, AD =

AB 2 BD 2

4a 2 a 2 =

3a 2 3a .

T-Ratios of 600
In ADB we have : ADB = 900 and ABD = 600.
Base = BC = a, Perp. = AD =

3a and Hyp. AB = 2a.

3a
3
AD

:
AB
2a
2
BD a 1
cos 600 =

:
AB 2a 2
3a
AD

3
tan 600 =
BD
a
1
1
2
1
1
cosec 600 =

; sec 600 =
2 ; cot 600 =

0
0
0
sin 60
tan 60
cos 60
3
3
sin 600 =

T-Ratios of 300
In ADB we have : ADB = 900 and ABD = 300.

Base = AD = 3a , Perp. = BD = a and Hyp. AB = 2a.

BD a 1

AB 2a 2
AD
3a

cos 300 =
2a
AB
BD
a
tan 300 =

AD
3a
1
cosec 300 =
2
sin 30 0
sin 300 =

3
:
2
1
3
; sec 300 =

1
1
2

; cot 300 =
3
0
tan 30 0
cos 30
3

T-Ratios of 00 and 900


T-Ratios of 00
We shall see what happens to the trigonometric ratios of angle A,
if it is made smaller and smaller in the right triangle ABC (see figure),
till it becomes zero. As A gets smaller and smaller, the length of the
side BC decreases. The point C gets closer to point B, and finally
when A becomes very close to 00, AC becomes almost the same as AB.

R E

-F O

U N D A T IO N

B Y

MANISH

MATHEMATICS

KUMAR

BC
is very close to O. Also
AC
AB
is very close to 1.
when A is very close to 00, AC is nearly same as AB and so the value of cos A =
AC

When A is very close to 00, BC gets very close to 0 and so the value of sin A =

This helps us to see how we can define the values of sin A cos A when A = 00. We define :
sin 00 = 0 and cos 00 = 1.
Using these, we have :

sin 0 0
0,
tan 0 =
cos 0 0
1
1

cot 00 =
0
tan 0
0
1
sec 00 =
1
cos 0 0
1
1
and cosec 00 =

0
sin 0
0
0

(not defined)

(not defined)

T-Ratios of 900
Now, we shall see what happens to the trigonometric ratios of A when it is made larger and larger in ABC
till itbecomes 900. As A gets larger and larger, C gets smaller and smaller. Therefore, as in the case above,
the length of the side AB goes on decreasing. The point A gets closer to point B. Finally when A is very close
900, C becomes very close to 00 and the side AC almost coincides with side BC (see figure).

R E

-F O

U N D A T IO N

B Y

MANISH

MATHEMATICS

KUMAR

When C is very close to 00, A is very close 900, side AC is nearly the same as side BC, and so sin A is very
close to 1. Also when A is very close to 900, C is very close 00, and the side AB is nearly zero, so cos A very
close to 0. So, we define:
sin 900 = 1 and cos 900 = 0.
Using these, we have :

sin 90 0 1
,
tan 90 =
cos 90 0 0
cos 90 0 1
0
cot 900 =
sin 90 0 0
1
1
cosec 900 =
1
0
sin 90
1
1
1

sec 900 =
cos 90 0 0

(not defined)

and

(not defined)

table for T-Ratios of Standard Angles


Angle
00
300
Ratio
Sin

cos

tan

cot

Not defined

sec

cosec

REMARK:

Not defined

1
2
3
2
1
3

450

600

1
2
1
2

3
2
1
2

1
3

2
3

2
3

900

1
0
Not
defined
0
Not
defined
1

(i)

As increases from 00 to 900, sin increases from 0 to 1.

(ii)
(iii)

As increases from 00 to 900, cos decreases from 1 to 0.


As increases from 00 to 900, tan increases from 0 to .

(iv)
(v)
(vi)
(vii)
(viii)

1
,0 0 90 0 is one.
sec
As cos decreases from 10 to0, increases from 0 to 900.
sin and cos can not be greater than one numerically.
sec and cosec can not be less than one numerically.
tan and cot can have any value.
The maximum value of

R E

-F O

U N D A T IO N

B Y

MANISH

MATHEMATICS

KUMAR

COMPETITION WINDOW
T-RATIOS OF SOME ANGLES LESS THAN 900
Angle

150

180

10
22
2

Ratio

Sin

Ex.7
Sol.

5 1
4

3 1
2 2

cos

3 1
2 2

10 2 5 1
2 2
4
2

tan

2 3

25 10 5
5

10 2 5
4
5 1
4
52 5

2 1

In ABC, right angled at B, BC = 5 cm, BAC = 300, find the length of the sides AB and AC.
We are given
BAC = 300, i.e., A = 300 and
BC = 5 cm
Now

BC
AC
AC = 2 5 or 10 cm
sinA =

or

sin 300 =

or
To find AB, we have,

Ex.8
Sol.

1
2 2
2

360

5
AC

or

5
1

AC 2

[ sin 300 =

1
]
2

AB
= cos A
AC
AB
= cos 300
or
10
3
3
AB
or
=
[ cos 300 =
]
2
AC
2
3

AB =
10 or 5 3 cm
2
Hence, AB = 5 3 cm and AC = 10 cm.
In ABC, right angled at C, if AC = 4 cm and AB = 8 cm. Find A and B.
We are given, AC = 4 cm and AB = 8 cm
Now sinB =

AC 4 1

AB 8 2

But we know that sin 300 =


Now A = 900 B

1
2

B = 300
[ A + B = 900]

= 900 300 = 600


Hence, A = 600 and B = 300.

10

R E

-F O

U N D A T IO N

B Y

MANISH

MATHEMATICS

KUMAR

Ex.9

Sol.

sin 30 0 tan 450 cos ec60 0


sec 30 0 cos 60 0 cot 450
1
2
1
0
0
0
sin 30 tan 45 cos ec60
2
3
=
0
0
0
2
1
sec 30 cos 60 cot 45
1
3 2

Evaluate :

[NCERT]

32 34
2 3
=
4 32 3
2 3
=

3 3 4 3 3 4 (3 3 4)

3 3 4 3 3 4 (3 3 4)

27 16 24 3 43 24 3

27 16
11

Ex.10 Find the value of in each of the following :


Sol.

(i) 2 sin 2 = 3
(i)
we have,

(ii) 2 cos 3 = 1

2 sin 2 =

sin 2 =

3
3
2

cos 3 =

1
2

cos 3 cos 600 3 600 = 200


we have,

(iii)

3 tan 2 3 = 0

sin 2 sin 600 2 600 = 300


we have,
2 cos 3 = 1

(ii)

(iii)

3 tan 2 3 = 0
3 tan 2 = 3
3
tan 2 =
3
3
tan 2 tan 600 2 600 = 300

COMPETION WINDOW

(i)
(ii)
(iii)

USING TRIGONOMETRIC TABLES


A Trigonometric Table consists of three parts :
A column on the extreme left containing degrees from 00 to 890.
Ten column headed by 0, 6, 12, 18, 24, 30, 36, 42, 48, and 54,
Five column of mean differences, headed by 1, 2, 3, 4, and 5, The mean differences is added in case of sines,
tangents and secants. The mean difference is subtracted in case of cosines, cotangents and cosecants.
The method of finding T-ratios of given angles using trigonometric tables, will be clear from the following
example :
Find the value of sin 430 52.

11

R E

-F O

U N D A T IO N

B Y

MANISH

MATHEMATICS

KUMAR

We have, 43052 = 43028 + 4


In the table natural sines, look at the numbers in the row against 430 and in the column headed 48 as shown
below.
From Table of
x0
0
6 12 18 24 30 36 42
48
54 1 2 3 4 5
Natural Sines :
Mean Differences
430 0.6820
0.6921
8
Now, sin 43048 = 0.6921
Mean difference for 4 = 0.0008

[To be added]
[See the number in the same row under 4]
sin 430 52 = [0.6921 + 0 0.0008] = 0.6929
TO FIND THE ANGLE WHEN ITS T-RATIOS IS GIVEN

Find , when sin = 0.7114.


From the table, find the angle whose sine is just smaller than 0.7114.
We have sin = 0.7114
Sin 450 18
= 0.7108
Diff.
= 0.0006
Mean difference of 6 corresponds to 3.

Required angle = (450 18 + 3) = 45021


Find , when cos = 0.5248
From the table, find the angle whose cosing is just smaller than 0.5248
We have cos = 0.5248
cos 580 18
= 0.5255
Diff.
= 0.0007
And 7 corresponds to 3.

Required angle = 580 18 + 3 = 580 21


TRY OUT THE FOLLOWING
1.
2.

Using tables find the value of :


(i) sin 830 12 (ii) cos 700 17
Using tables find the value of if :
(i) sin = 0.42 (ii) cos = 0.8092
(vi) cot = 0.1385

(iii) tan 240 14 (iv) cosec (30.8)0

(v) sec 680 10 (vi) cot 390 15

(iii) tan = 2.91 (iv) cosec = 2.8893

(v) sec = 1.2304

ANSWERS
1.
2.

(i) 0.993.
(i) 240 50

(ii) 0.3373
(ii) 350 59

(iii) 0.4536
(iii) 710 2

(iv) 1.9530
(iv) 200 15

(v) 2.6892
(v) 350 38

(vi) 1.2283
(vi) 820 7

12

R E

-F O

U N D A T IO N

B Y

MANISH

MATHEMATICS

KUMAR

T-RATIOS OF COMPLEMENTARY ANGLES


Complementary Angles
Two angles are said to be complementary, if their sum is 900.
Thus, 0 and (900 ) are complementary angles.
T-ratios of Complementary Angles
Consider ABC in which B = 900 and A = 0.

C = (900 ).
Let AB = x. BC = y and AC = r.
When we consider the T-ratios of (900 ), then
Base = BC, Perp. = AB and Hyp. AC =

AB x
= cos .
AC r
BC y
cos (900 ) =
= sin .
AC r
AB x
tan (900 ) =
= cot .
BC y
1
1
cosec (900 ) =

= sec .
0
sin(90 ) sin
1
1

= cosec .
sec (900 ) =
0
cos(90 ) sin
1
1

= tan .
cot (900 ) =
0
tan(90 ) cot
sin (900 ) =

(i) sin (900 ) = cos


(iv) cosec (900 ) = sec

(ii) cos (900 ) = sin


(v) sec (900 ) = cosec

(iii) tan (900 ) = cot


(vi) cot (900 ) = tan

Aid to memory
Add co if that is not there
Remove co if that is there
Thus we have,
sine of (900 ) = cosine of sine (900 ) = cos
cosine of (900 ) = sine of cos (900 ) = sin
tangent of (900 ) = cotangent of tan (900 ) = cot
cotangent of (900 ) = tangent of cot (900 ) = tan
secant of (900 ) = cosecant of sec (900 ) = cosec
cosecant of (900 ) = secant of cosec (900 ) = sec
In other words :
sin (angle) = cos (complement) ;
cos (angle) = sin (complement)
tan (angle) = cot (complement) ;
cot (angle) = tan (complement)
sec (angle) = cosec (complement) ;
cosec (angle) = sec (complement)
where complement = 900 angle

13

R E

-F O

U N D A T IO N

B Y

MANISH

MATHEMATICS

KUMAR

Ex.11 Without using tables, evaluate :

Sol.

(i)

sin 530
cos 37 0

(i)

sin 530 sin(90 0 37 0 ) cos 37 0

1
cos 37 0
cos 37 0
cos 37 0

(ii)

cos 49 0
sin 410

(iii)

tan 66 0
cot 24 0

cos 530 cos(90 0 410 ) sin 410

1
sin 37 0
sin 410
sin 410
tan 66 0 tan(90 0 24 0 ) cot 24 0

1
(iii)
cot 24 0
cot 24 0
cot 24 0
(ii)

REMARK :

(i)
(ii)

[ sin (900 ) cos ]


[ cos (900 ) sin ]
[ tan (900 ) cot ]

The above example suggests that out of the two t-ratios, we convent one is term of the
t-ratios of the complement.
For uniformity, we usually convert the angle greater than 450 in terms of its complement.

Ex.12 Without using tables, show that (cos 350 cos 550 sin 350 sin 550) = 0.
Sol.
LHS = (cos 350 cos 550 sin 350 sin 550)
= [(cos 350 cos 550 sin (900 550) sin (900 350)]
= (cos 350 cos 550 cos 550 cos 350) = 0 = RHS.
[ sin (900 ) cos and cos (900 ) sin ]
Ex.13 Express (sin 580 + cosec 850) in terms of trigonometric ratios of angles between 00 and 450.
Sol.
(sin 580 + cosec 850) = sin (900 50) cosec (900 50) = (cos 50 + sec 50).
Ex.14 If tan 2A = cot (A 180), where 2A is an acute angle, find the value of A.
Sol.
We are given,
tan 2A = cot (A 180)
or
cot (900 2A) = cot (A 180)
[ cot (900 2A) = tan 2A]

900 2A = A 180
or
A + 2A = 900 180
or
3A = 1080

A = 360
Ex.15 Evaluate :
Sol.

sec 29 0
2 cot 80 cot 17 0 cot 450 cot 730 cot 82 0 .
0
cos ec61

sec 29 0
2 cot 80 cot 17 0 cot 450 cot 730 cot 82 0 .
cos ec610
=

sec 29 0
2 cot 8 0 cot 17 0 (1) cot(90 0 17 0 ) cot(90 0 8 0 )
0
0
cos ec(90 29 )

sec 29 0
2 cot 80 cot 17 0 tan 17 0 tan 80 .
sec 29 0
1
1
= 1 2 cot 8 0 cot 17 0.
.
0
cot 17 cot 80

[ cot (900 ) = tan ]

tan

cot
14

R E

-F O

U N D A T IO N

B Y

MANISH

MATHEMATICS

KUMAR

A
BC
cos , where A, B and C are interior angles of ABC.
2
2

Ex.16 For a triangle ABC, show that sin


Sol.

We know that A + B + C = 1800


Thus we have, B + C = 1800 A
or

A
BC
0
90

2
2

0 A
BC
sin
sin 90 or
2

or

A
BC
sin
cos .
2
2

T-IDENTITIES
We know that an equation is called an identity when it is true for all value of the variables involved. Similarly, an
equation involving trigonometric ratios of an angle is called a trigonometric identity, if it is true for all
values of the angle(s) the angle(s) involved.
The three Fundamental Trigonometric Identities are
A
(i)
cos2 A + sin2 A = 1 ; 00 A 900
2
2
0
0
(ii)
1 + tan A + cosec A = 1 ; 0 A < 90
(iii)
1 + cot2 A + cosec2 A = 1 ; 00 < A 900

(i)

Geometrical Proof :
Consider a ABC, right angled at B. Then we have :
AB2 + BC2 = AC2
(i) By Pythagoras theorem
2
2
0
cos A + sin A = 1 ; 0 A 900
Dividing each term of (i) by AC2, we get

AB 2 BC 2 AC 2

AC 2 AC 2 AC 2
2

i.e.,

AB BC AC

AC AC AC

(cos A)2 + (sin A)2 = 1


(ii)
cos2 A + sin2 A = 1
0
This is true for all A such that 0 A 900
So, this is a trigonometric identity.
1 + tan2 A = sec2 A ; 00 A < 900
Let us now divide (i) by AB2. We get
i.e.,
i.e.,

(ii)

AB 2 BC 2 AC 2

AB 2 AB 2 AB 2
2

or,

AC
AB BC

AB
AB AB

i.e.,
1 + tan2 A = sec2 A
(iii)
0
This equation is true for A = 0 . Since tan A and sec A are not defined for A = 900, so (iii) is true for all A such
that 00 A < 900

15

R E

-F O

U N D A T IO N

B Y

MANISH

MATHEMATICS

KUMAR

(iii)

1 + cot2 A = cosec2 A : 00 < A < 900


Again, let us divide (i) by BC2, we get

AB 2 BC 2 AC 2

BC 2 BC 2 BC 2

AB BC AC

BC BC BC

1 + cot2 A = cosec2 A
(iii)
Since cosec A and cot A are not defined for A = 00, therefore (iv) is true for all A such that : 00 < A < 900
Using the above trigonometric identities, we can express each trigonometric ratio in terms of the other
rigonometric ratios, i.e., if any one of the ratios is known, we can also determine the value of other trigonometric
ratios.
Fundamental Identities (Results)

sin 2 cos 2 1
sin 2 1 cos 2
cos 2 1 sin 2

1 tan 2 sec 2
sec 2 tan 2 1
tan 2 sec 2 1

1 cot 2 cos ec 2
cos ec 2 cot 2 1
cot 2 cos ec 2 1

To prove Trigonometrical Identities


The following methods are to be followed :
Method-I :
Table the more complicated side of the identity (L.H.S. or R.H.S. as the case may be) and by
using suitable trigonometric and algebraic formula prove it equal to the other side.
Method-II :
When neither side of the identity is in a simple form, simplify the L.H.S. and R.H.S. separately by
using suitable formulae (by expressing all the T-ratios occurring in the identity in terms of the
sine and cosine and show that the results are equal).
Method-III : If the identity to the proved is true, transposing so as to get similar terms on the same side, or
cross-multiplication, and using suitable formulae, we get an identity which is true.
Ex.17 Express sin A, sec A and tan A in terms of cot A.
Sol.
We know that

[NCERT]

1
1
1

cos ec A
cos ec 2 A
1 cot 2 A
1
1
sin A
(Dividing num. and deno, by sin A)
sec A =

cos A cos A
sin A

sin A

1
cos ec A
1 cot 2 A
and tan A

cot A
cot A
cot A

Ex.18 Prove
Sol.

LHS

sec 2 cos ec 2 tan cot


=

sec 2 cos ec 2 (1 tan 2 ) (1 cot 2 ) tan 2 cot 2 2 tan cot

= tan cot tan cot = RHS


Hence, proved.

16

R E

-F O

U N D A T IO N

B Y

MANISH

MATHEMATICS

KUMAR

1
tan A cot A
1

sin A
= (cos ecA sin A)(sec A cos A)
sin A

Ex.19 Prove (cos ecA sin A)(sec A cos A)


Sol.

LHS

1 sin 2 A 1 cos 2 A cos 2 A sin 2 A

cos A sin A cos A


sin A

sin A cos A
sin 2 A cos 2 A
sin A cos A
sin A cos A
=
2
sin A
cos 2 A

sin A cos A sin A cos A


1
= RHS
=
tan A cot A

cos A

cos A

[ sin 2 A cos 2 A 1 ]
[ sin 2 A cos 2 A 1 ]

= sin A cos A

[Dividing the numerator and denominator by sin A cos A.]

Hence, proved.

APPLICATIONS OF TRIGONOMETRY
Many times, we have to find the height and distances of many objects in real life. We use trigonometry to solve
problems, such as finding the height of a tower, height of a flagmast, distance between two objects, where
measuring directly is trouble, some and some times impossible. In those cases, we adopt indirect methods which
involves solution of right triangles.
Thus Trigonometry is very useful in geography, astronomy and navigation. It helps us to prepare maps, determine
the position of a landmass in relation to the longitudes and latitudes. Surveyors have made use of this knowledge
since ages.
Angle of Elevation
The angle between the horizontal line drawn through the observer eye and line joining the eye to any object
is called the angle of elevation of the object, if the object is at a higher level than the eye i.e., If a horizontal line
OX is drawn through O, the eye of the observer, and P is an object in the vertical plane through OX, then if P is
above OX, as in fig. XOP is called the angle of elevation or the altitude of P as seen from O.

Angle of Depression
The angle between the horizontal line drawn through the observer eye and line joining the eye to any object
is called the angle of depression of the object, if the object is at a higher level
than the eye i.e., If a horizontal line OX is drawn through O, the eye of the
observer, and P is an object in the vertical plane through OX, then if P is above
OX, as in fig. XOP is called the angle of depression of P as seen from O.

17

R E

-F O

U N D A T IO N

B Y

MANISH

MATHEMATICS

KUMAR

REMARK :

1.
2.
3.
4.
5.

The angle of elevation as well as angle of depression are measured with reference to horizontal line.
All objects such as towers, mountains etc. shall be considered as linear for mathematical convenience,
throughout this section.
The height of the observer, is neglected, if it is not given in the problem.
Angle of depression of P as seen from O is equal to the angle of elevation of O, as seen from P.
i.e., AOP = OPX.
To find one side a right angled triangle when another side and an acute angle are given, the hypotenuse also
being regarded as a side.

Re quired
= a certain T-ratio of the given angle.
Given side
6.
7.

The angle of elevation increases as the object moves towards the right of the line of sight.
The angle of depression increases as the object moves towards the right of the line of sight.

COMPWRIRION WINDOW
BEARING OF A POINT

The true bearing to a point is the angle measured in degrees in a clockwise direction from the north line. We will
refer to the true bearing simply as the bearing.

e.g. (i) the bearing of point P is 650


(ii) the bearing of point Q is 3000
A bearing is used to represent the direction of one point relative to another point.
e.g., the bearing of A from B is 600. The bearing of B from A is 2400.

18

R E

-F O

U N D A T IO N

B Y

MANISH

MATHEMATICS

KUMAR

TRY OUT THE FOLLOWING


State the bearing of the point P in each of the following diagrams :

ANSWERS
(a) 480

(b) 2400

(c) 1400

(d) 2900

Ex.20 An observer 1.5 m tall, is 28 5 m away from a tower 30 m high. Determine the angle of elevation of the top of the
tower from his eye.
(NCERT)
Sol.
Let AB be the height of the tower, CD the height of the observer with his eye at the point D, AB = 30 m, CD = 1.5
m.
Through D, draw DECA than BDE = where is the angle of elevation of the top of the tower from his eye.
AC = horizontal distance between the tower and the observer = 28.5 m
BE = AB AE = (30 1.5) m = 28.5 m BDE is right triangle at E,

28.5
BE
tan
tan tan 1
28.5
DE
tan 1 tan 450 450 .

then

Required angle of elevation of the tower = = 450.


Ex.21 A vertical post casts a shadow 21 m long when the altitude of the sun is 300. Find :
(a)
the height of the post.
(b)
the length of the shadow when the altitude of the sun is 600.
Sol.

(c)
the altitude of the sun when the length of the shadow is 7 3 m.
Let AB be the vertical post and its shadow is 21 m when the altitude of the sun is 300.
(a)
BC = 21 m, ACB = 300, AB = h metres
ABC is rt. ,

AB
h
1
tan 30 0

BC
21
3

21 7 3 3

7 3 AB = h, Height of the pole = 7 3


3
3

(b)

In this case, we have,


ACB = 600, BC = x m AB = 7 3 m
ABC is rt. , then :

AB
tan 60 0 3
BC
h
7 3
3
3
x
x
x = BC, Length of the shadow = 7 m.

19

R E

-F O

U N D A T IO N

B Y

MANISH

MATHEMATICS

KUMAR

(c)

In this case :
AB = h = 7 3
BC = The length of the shadow = 7 3 m
when the altitude of the sun is

AB
7 3
tan
tan
BC
7 3
tan = 1 = tan 450 450

ABC is rt. , then

Altitude of the sun = = 450


Ex.22 A 1.6 m tall girl stands at a distance of 3.2 m from the lamp-post and casts a shadow of 4.8 m on the ground. Find
the height of the lamp-post by using (i) trigonometric ratios (ii) property of similar trianges. (NCERT)
Sol.
Let PQ be the position of the lamp-post whose height is h metres. i.e., PQ = h metres. AB be the position of the
tall girl such that AB = 16. m. Let BC be the shadow of AB such that BC = 4.8 m, ACB = PAE =
(corr. s)

(i)

In right ABC,

AB
1.6
1
tan
tan tan
BC
4.8
3
AB = EQ = 1.6 m. Also AE = BQ = 3.2 m
PE = PQ EQ = (h 1.6) m

PE
h 1.6 1
(h 1.6)
1
tan
tan

AE
3.2
3
3.2
3
3h 4.8 = 3.2 3h = 4.8 + 3.2 = 8 3h = 8
2
8
The height of the lamp-post = m = 2 m
3
3
In two s ACB and PCQ, we have :
CQ = CB + BQ
ACB = ACB = (common)
= (4.8 + 3.2) m = 8 metres
ABC = PQC = 900 ACB ~ PAE
(AA simila)
AC CB AB
BC AB

PC CQ PQ
CQ PQ
BC AB
4.8 1.6
3 1

3h 8
CQ PQ
h
8
8 h
3
2
Thus 3h 8 h m h 2 m
8
3
2
Required height of the lamp-post = PQ = h = 2 m
3
In right ABC,

(ii)

20

R E

-F O

U N D A T IO N

B Y

MANISH

MATHEMATICS

KUMAR

Ex.23 A captain of an airplane flying at an altitude of 1000 metres sights two ships as shown in the figure. If the angle of
depressions are 600 and 300, find the distance between the ships.

Sol.

Let A be the position of the captain of an airplane flying at the altitude of 1000 metres from the ground.
AB = the altitude of the airplane from the ground = 1000 m
P and Q be the position of two ships.
Let
PB = x metres, and BQ = y metres.
Required :
PQ = Distance between the ships = (x + y) metres.
ABP is rt. at B
ABQ is rt. at B

AB
tan 60 0
PB
1000
1000
3x
x
3
1000(1.732)
x
577.3 m
3

AB
tan 60 0
BQ
1000
1

y 1000 3
y
3

Required distance between the ships = (x + y) metres = (577.3 + 1732) m = 2309.3 m


Ex.24 Two poles of equal heights are standing opposite to each other on either side of a road, which is 80 metres wide.
From a point between them on the road, the angles of elevation of their top are 300 and 600. Find the position of
the point and also the height of the poles.
Sol.
Let AB and CD be two poles of equal height standing opposite to each of their on either side of the road BD.

AB = CD = h metres.
Let P be the observation point on the road BD. The angles of elevation of their top are 300 and 600.
APB = 300, CPD = 600
The width of the road = BD = 80 m, let PD = x metres
Then BP = (80 - x) metres
Consider right CDP, we have :

CD
h
tan 60 0 3 h 3 x
(i)
PD
x
In right ABP, we have :
AB
h
1
80 x

h
(ii)
tan 30 0
BP
80 x
3
3
h 3x
(80 x) 3x 4 x 80 x 20
From (i) and (ii), we get :
80 x
h
3
Height of each pole = AB = CD = 3 . x = 20. 3 = 20 (1.732) = 34.64 metres.
Position of point P is 20 m from the first and 60 m from the second pole.
i.e., position of the point P is 20 m from either of the poles.

21

R E

-F O

U N D A T IO N

B Y

MANISH

MATHEMATICS

KUMAR

SYNOPSIS

1.

In a right triangle ABC, with right angle B,

sin A

Perpendicular
Hypotenuse

cos A

Hypotenuse
Perpendicular

cos A

Base
Hypotenuse

sec A

Hypotenuse
Base

tan A

Perpendicular
Base

cot A

Base
Perpendicular

1
1
1
: sec A
: tan A
sin A
cos A
cot A

2.

cos

3.

The value of sinA or cosA never exceeds 1, whereas the value of secA or cosecA is always greater than or equal
to 1.

4.

sin (90 0 A) cos A

cot (90 0 A) tan A

cos (90 0 A) sin A

sec (90 0 A) cos ecA

tan (90 0 A) cot A

cos ec (90 0 A) sec A

5.

sin 2 A cos 2 A 1 : 1 tan 2 A sec 2 : 1 cot 2 A cos ec 2 A

6.

If one of the sides and any other part (either an acute angle or any side) of a right triangle is known, the remaining
sides and angles of the triangle can be easily determined.

7.

In a right triangle, the side opposite to 300 is half the side of the hypotenuse.

8.

In a right triangle, the side opposite to 600 is

9.

(i)

3
times the side of the hypotenuse.
2

The line of sight is the line drawn from the eye of an observer to the point in the object viewed by the
observer.

(ii)

The angle of elevation of an object viewed is the angle formed by the line of sight with the horizontal
when it is above the horizontal level i.e., the case when we raise our head to look at the object.

(iii)

The angle of depression of an object viewed is the angle formed by the line of sight with the horizontal
when it is above the horizontal level i.e., the case when we raise our head to look at the object.

22

R E

-F O

U N D A T IO N

B Y

MANISH

MATHEMATICS

KUMAR

EXERCISE 1

(FOR SCHOOL/BOARD EXAMS)


OBJECTIVE TYPE QUESTIONS

1.

CHOOSE THE CORRECT ONE


In ABC , B 90 0 . If AB = 14 cm and AC = 50 cm then tan A equals :
(A)

2.

3.

4.

5.

24
25

24
7

(C)

7
24

12
2 cos 3 tan
then the value of the
is :
13
sin tan sin
5
12
259
(A)
(B)
(C)
5
102
3
2
2
p q
p sin q cos
If sec
then the value of the
is :
q
p sin q cos
p
p2
p2 q2
(B) 2
(C) 2
(A)
q
q
p q2
8
If angle A is acute and cos A =
then cot A is :
17
8
17
15
(B)
(C)
(A)
15
8
8
sec is equal to
1

1 cot 2
cot

(B)

1 cos 2 0

sin 30 + cos 60 equals :


(A)

1 3
2

(C)

(B)
2

cot
1 cot 2

(C) 1
0

The value of 2 tan 60 4 cos 45 3 sec 30 is :


(A) 0
(B) 1
(C) 12

8.

3
The value of
tan2 300 3 sin2 600 + 3 cosec2 450 is
4

(A) 1
(B) 8
7 sin2 + 3 cos2 = 4 then :
(A) tan

10.
11.
12.

25
24

(D)

259
65

(D)

p2 q2
p2 q2

(D)

17
15

1
2

(B) tan

(D) None of these


(D) 8

(C) 0

1
2

The solution of the trigonometric equation

(C) tan

(D) 12

1
3

(B)

3
2

(D) tan

cos 2
3,0 0 90 0 :
2
2
cot cos 0
(C) = 60
(D) = 900

(B) = 300
(A) = 00
If cot + cos = p and cot = q, then the value of p2 q2 is :
(B) 4 pq
(C) 2 pq
(A) 2 pq
The value of sin2 150 + sin2 300 + sin2 450 + sin2 600 + sin2 750 is :
(A) 1

(D)

cos ec 2 1
cos ec

7.

9.

(D)

If sin

(A)
6.

(B)

(C)

5
2

1
3

(D) 4 pq
(D) 3

23

R E

-F O

U N D A T IO N

B Y

MANISH

MATHEMATICS

KUMAR

13.

sin 29 0 sin 610


The value of
is :

cos 610 cos 29 0


(A) Zero

14.
15.

(B) 1

17.
18.
19.

(B) Zero

21.

22.

23.

24.

25.

29
61 0

(C)

1
2

(D) 2

A flagstaff 6 metres high throws shadow 2 3 metres long on the ground. The angle of elevation is :
(B) 450
(C) 900
(D) 600
(A) 300
An observer 3 m tall is 3 m away from the pole 2 3 m high. The angle of the top of elevation of the top from
the pole is :
(A) 450
(B) 300
(C) 600
(D) 150
An observer 1.5 m tall is 28.5 m away from. a chimney. The angle of elevation of the top of the chimney from her
eyes is 450. The height of the chimney is :
(A) 30 m
(B) 27 m
(C) 28.5 m
(D) None of these
The angle of elevation of the top of a tower from a distance 100 m from its foot is 300. The height of the tower is :
(A) 1000 3 m

20.

(D)

The values of x and y which make the following solutions true are: cosx0 = sin 52 and cos y0 = sin (y0 + 10)
(A) x = 520, y = 300
(B) x = 380, y = 400
(C) x = 480, y = 520
(D) x = 400, y = 500
If 90 0 and 2 then cos 2 sin 2 equal :
(A) 1

16.

61
29

(C)

(B)

200
m
3

(C) 5 3 m

(D)

100
m
3

A kite is flying at a height of 60 m above the ground. The sting attached to the kite is temporarily the to a point on
the ground. The inclination of the string with the ground is 600. The length of the staring is :
(B) 30 m
(C) 20 3 m
(D) 60 3 m
(A) 40 3 m
A tree is broken by the wind. Its top struck the ground at an angle 300 at a distance of 30 m from its foot. The
whole height of the tree is :
(B) 20 3 m
(C) 40 3 m
(D) 30 3 m
(A) 10 3 m
From a point on a bridge across a river, the angles of depression of the banks on opposite sides of the river are 300
and 450 respectively. If the bridge is at a height of 3 m from the banks then the width of the river is :
(A) 3 ( 3 1) m
(B) 3 ( 3 1) m
(C) ( 3 3) m
(D) ( 3 3) m
The angles of elevation of the top of a tower from two points at a distance of 4 m and 9 m from the base of the
tower and in the same straight line with it are complementary. The height of the tower is :
(B) 13 m
(C) 6 m
(D) 2.25 m
(A) 5 m
A 1.5 m tall boy is standing at some distance from a 30 m tall building. The angles of elevation from his eyes to
the top of the building increases from 30 to 600 as he walks towards the building. The distance he walked towards
the building is :
(B) 57 3 m
(C) 38 3 m
(D) 18 3 m
(A) 19 3 m
As observed from the top of a 75 m high lighthouse from the sea-level, the angles of depression of two ships are
300 and 600. if one strip is exactly behind the other on the same side of the light-house then the distance between
the two ships is :
(B) 75 3 m
(C) 50 3 m
(D) None of these
(A) 25 3 m
(OBJECTIVE)

Que.
Ans.
Que.
Ans.
Que.
Ans.

1
B
11
B
2
D

EXERCISE

ANSWER KEY
2
C
12
C
122
B

3
C
13
A
23
C

4
A
14
B
24
A

5
B
15
C
25
C

6
C
16
D

7
A
17
B

8
C
18
A

9
D
19
D

10
C
20
A

24

R E

-F O

U N D A T IO N

B Y

MANISH

MATHEMATICS

KUMAR

EXERCISE 2

(FOR SCHOOL/BOARD EXAMS)


SUBJECTIVE TYPE QUESTIONS

VERY SHORT ANSWER TYPE QUESTIONS


1.

In the adjoining fig, determine : sin sin

2.

If 5 tan = 4, find the value of

3.

If A = 300. verify sin 2A = 2 sin A cos A :

4.
5.
6.
7.

8.
9.
10.
11.
12.

5 sin 3 cos
:
sin cos

1
cos ec 2 sec 2
, what is the value of
?
cos ec 2 sec 2
5
1
What is the maximum value of
?
sec
tan
What is the value of if sin cos
2
In the given fig ABC is right at B such that AB = 3 cm and AC = 6 cm. Determine ACB.
Given that tan

2 tan 30 0
1 tan 2 30 0
cos
sin
Evaluate :

0
sin(90 ) cos(90 0 )
Evaluate : sin 250 cos 650 cos 250 sin 650
3
If tan A =
and A + B = 900, then what is the value of cot B?
4
0
Evaluate :

If tan A = cot B, prove that A + B = 90

2 tan 80 0
3 cot 10 0

13.

Evaluate :

14.
15.

The height of a tower is 10 m. Calculate the height of its shadow when suns altitude is 450.
What is the angle of elevation of the sum when the length of the shadow of a pole is 3 times of the height of the
pole ?

25

R E

-F O

U N D A T IO N

B Y

MANISH

MATHEMATICS

KUMAR

SHORT ANSWER TYPE QUESTIONS


1.
2.

3.
4.
5.
6.

3
5 sin A 3 sec A 3 tan A
. evaluate
5
4 cot A 4 cos ecA 5 cos A
q
cos ec cot
Given cos
, find
cos ec cot
p2 q2

If cos A =

4
12
and cos
find sin cos cos sin
5
13
1
1
, prove that sec tan 2 x or
If sec x
.
4x
2x
1 cot
1 cos 16
If
, find
.
1 cot
1 cos 9
cos ec sec
sec tan 36
If

, find
.
sec tan 49
cos ec sec
If sin

7.

If 2 sin + cos = 2, find sin

8.

If

9.

(i)

10.
11.

(ii)
If tan (2A + B) = 3 and cot (3A B) = 3 , find A and B.
Find x if : (i) cos (5x 400) = sin 300. (ii) cosec (x + 300) = cot 450.
If A = 600. B = 300, verify each of the following :

12.

1 sin x
1 cos x
7 4 3 . find the value of
.
1 sin x
1 cos x
If sin (A + B) = 1 and cos (A B) = 1, find A and B.

(ii)

cot A cot B 1
cot A cot B

cos (A - B) cos A cos B + sin A sin B.

(i)

Assume that tan (A + B) =

(ii)

Assume cos (A - B) = cos A cos B + sin A sin B, find cos 150 when A = 450, B = 300.

tan A tan B
, find tan 750 when A = 450, B = 300.
1 tan A tan B

13.

Assume that sin (A + B) = cos A cos B + cos A sin B, if sin A =

14.

Find the value of in each of the following if :


(i) 2 cos 3 = 1

15.
16.

cos (A + B)

(i)

(ii) 2 3 tan = 6

(iii)

1
1
, sin B =
. Then find angle (A + B)
5
10

1 tan 2 1

1 tan 2 2

Prove that : (i) sin 600 cos 300 + cos 600 sin 300 = sin 900 (ii) cos 900 = 4 cos3cos 300 3 cos 300
Evaluate the following :

3 2 0
3
cot 30 3 sin 2 60 0 2 cos ec 2 60 0 tan 2 30 0
4
4
2
0
1 cot 2 60 0
1 tan 30
2
0
2
0
2
0
(ii)

ec

cos
60
cos
45
sin
45
1 cot 2 60 0
1 tan 2 30 0
1
(iii) cos 2 30 0 cos 2 450 4 sec 2 60 0 cos 2 90 0 2 tan 2 60 0
2
4
0
4
0
2
(iv) 4(sin 30 cos 60 ) 3(cos 45 0 sin 2 90 0 )

(i)

26

R E

-F O

U N D A T IO N

B Y

MANISH

17.

KUMAR
Find the value of x in each of the following :

MATHEMATICS

27 cos 2 450
.
(i) 2 x tan 60 3 x sin 30
4 sin 2 60 0
(ii) ( x 1)(sin 4 60 0 cos 4 30 0 ) x(tan 2 60 0 tan 2 450 ) ( x 2) cos 2 450 1 .
(iii) ( x 4) sin 2 60 0 ( x 5) tan 2 30 0 x sin 45 0 cos 450 0 .
(iv) tan x sin 45 0 cos 45 0 sin 30 0 .
(v) sin 2 x sin 60 0 cos 30 0 cos 60 0 sin 30 0 .
(vi) tan 3 x sin 450 cos 450 sin 30 0 .
2

18.

If (sec x 1) (sec x + 1) = 3 then find the value of x.

19.

tan 3
cot 3

sec cos ec 2 sin cos


Prove :
1 tan 2 1 cot 2

20.

Prove : (tan cot sec) (tan cot sec ) cos ec 2 .

21.

Prove : (sec 2 A tan 2 A) (cos ec 2 A cot 2 A) 1 2 sec 2 A cos ec 2 A .


2

22.

Prove

23.

Prove

24.
25.
26.
27.

Prove
Prove
Prove
Prove

28.

Prove

29.

Prove

30.

Prove

31.

Prove

32.

Prove

33.

Prove

34.

Prove

35.

Prove

36.

Prove

37.

Prove

38.

Prove

1 cot 2 1 cot

1 tan 2 1 tan
tan
cot

sin cos .
:
2
2
(1 tan )
(1 cot 2 )
: (1 cos sin ) (1 cos sin ) 2 sin (1 sin )
: sin A (1 tan A) cos A (1 cot A) sec A cos ecA .
: sec (1 sin ) (sec tan ) 1
: sec (1 sin ) (sec tan ) 1
: cos 2 (1 tan 2 ) sin 2 (1 cot 2 ) 2
cot
:
1
3
cos cos ec sin cos
: (sec cos ec ) (sin cos ) sec cos ec 2
2(1 sin 2 )
: (tan sec ) 2 (tan sec ) 2
cos 2
(sin cos ) 2 (sin cos ) 2
2 sin cos
:
(sin cos ) 2 (sin cos ) 2
: (1 sin cos ) 2 2 (1 sin ) (1 cos )
: sin 8 cec 8 (sin 2 cos 2 ) (1 2 sin 2 cos 2 )
: sec 6 tan 6 1 3 sec 2 tan 2
: sec 4 tan 4 2 sec 2 1
cos 3 sin 3 cos 3 sin 3
:

2 sin cos
cos sin
cos sin
sin A cos A sin A cos A
2
:

2
sin A cos A sin A cos A sin A cos 2 A

27

R E

-F O

U N D A T IO N

B Y

MANISH

MATHEMATICS

KUMAR

39.

Prove :

40.

Prove :

41.

Prove :

42.

Prove :

43.

Prove :

44.

Prove :

45.

Prove :

46.

Prove :

47.

Prove :

48.

Prove :

49.

Prove :

50.

Prove :

51.

Prove :

52.

Prove :

53.

Prove :

54.

Prove :

55.

Prove :

56.

Prove :

57.

Prove :

1
1

2 sec 2
1 sin 1 sin
1
1

2 sec
sec tan sec tan
cos
cos

2 sec
1 sin 1 sin
1
1
1
1

cos ec cot sin sin cos ec cot


1 cos
sin

2 cos ec
sin
1 cos
sin cos
sin cos
2
2 sec 2

sin cos sin cos sin 2 cos 2 tan 2 1


sin A sin B cos A cos B

0
cos A cos B sin sin B
1 cot 1 cot
2

2
1 cot 1 cot sin cos 2
cos ec 1
cos ec 1

2 sec
cos ec 1
cos ec 1
sec 1 tan tan sec 1
2 sec

sec 1 tan tan sec 1


1 cos
cos ec cot
1 cos
1 sin
cot 2

1 sin
(cos ec 1) 2
sec 1
tan

sec 1
sec 1
sec tan
sec tan
sec tan
1 sin
1 2 tan 2 2 tan sec
1 sin
cos ec cot
1 2 cot 2 2 cos ec cot
cos ec cot
cos ec cot
1

cos ec cot
cos ec cot
sec 1
sec 1

2 cos ec
sec 1
sec 1
cos ec cot
cos ec cot

2 cos ec
cos ec cot
cos ec cot

28

R E

-F O

U N D A T IO N

B Y

MANISH

MATHEMATICS

KUMAR

58.
59.
60.
61.
62.
63.
64.
65.
66.
67.

68.

tan A sec A 1
1 sin A 1 sin A
sec A tan A
tan A sec A 1
cos A 1 sin A
1 cos A sin A
1 sin A 1 sin A
Prove :
sec A tan A
1 cos A sin A
cos A 1 sin A
cos A 1 sin A
1 cos A 1 cos A
Prove :
cos ecA cot A
cos A 1 sin A
sin A 1 cos A
What is the angle of elevation of a vertical flagstaff of height 100 3 m from a point 100 m from its foot.
Prove :

A ladder makes an angle of 600 with the floor and its lower end is 20 m from the wall. Find the length of the
ladder.
The shadow of a building is 100 m long when the angle of elevation of the sun is 600. Find the height of the
building.
A ladder 20 m long is placed against a vertical wall of height 10 metres. Find the distance between the foot of the
ladder and the wall and also the inclination of the ladder to the horizontal.
What is the angle of elevation of the sun when the length of the shadow of the pole is

1
times the height of the
3

pole ?
A flagstaff 6 metres high throws a shadow 2 3 metres along on the ground. Find the angle of elevation of the
sun.
A tree 10 (2 3 ) metres high is broken by the wind at a height 10 3 metres from its root in shch a way that top
struck the ground at certain angle and horizontal distance from the root of the tree to the point where the top meets
the ground is 10 m. Find the angle of elevation made by the top of the tree with the ground.
A tree is broken at certain height and its upper part 9 2 m long not completely separated meet the ground at an
angle of 450. Find the height of the tree before it was broken and also find the distance from the root of the tree to
the point where the top of the tree meets the ground.

LONG ANSWER TYPE QUESTIONS


1.
2.
3.

The ladder resting against a vertical wall is inclined at an angle of 300 to the ground. The foot of the ladder is 7.5
m from the wall. Find the length of the ladder.
A circus artist is climbing a 20 m long rope, which is tightly stretched and tied from the top of vertical pole to the
ground. Find the height of the pole if the angle made by the rope with the ground level is 300.
The length of a string between a kite and a point on the roof of a building 10 m high is 180 m. If the string makes
an angle with the level ground such that tan

4.
5.
6.
7.
8.
9.

4
, how high is the kite from the ground?
3
0

The angle of depression of a ship as seen from the top of 120 m high light house is 60 . How far is the ship from
the light house?
A boy 1.7 m tall, is 25 m away from a tower and observes the angle of elevation of the top of the tower to be 600.
Find the height of the tower.
A man 1.8 m tall stands at a distance of 3.6 m from a lamp post and casts a shadow of 5.4 m on the ground. Find
the height of the tower.
A straight highway leads to the foot of a tower of height 50 m. from the top of the tower, the angles of depression
of two cars standing on the highway are 300 and 600. What is the distance between the two cars and how far is
each car from the tower ?
Two points A and B are on opposite sides of a tower. The top of the tower makes an angle of 300 and 450 at A
and B respectively. If the height of the tower is 40 metres, find the distance AB.
Two men on either side of a tower 60 metres high observe the angle of elevation of the top of the tower to be 450
and 600 respectively. Find the distance between the two men.

29

R E

-F O

U N D A T IO N

B Y

MANISH

10.

MATHEMATICS

KUMAR
Two boats approach a light house in the middle of the sea from opposite directions. The angles of elevation of the
top of the light house from two boats are and . If the distance between the two boats is x metres, prove that
the height of the light house is

x
cot cot
(i) Find h if 60 0 , 45 0 and x = 250 m
h

11.

12.

13.

14.

15.
16.
17.

(ii) Find h if 60 0 , 30 0 and x = 400 m


A boy standing on a horizontal plane finds a bird flying a distance of 100 m from him at an elevation of 300. A
girl standing on the roof of 20 metres high building, finds the angle of elevation of the same bird to be 450. Both
the boy and the girl are on opposite sides of the bird. Find the distance of the bird from the girl.
Two pillars of equal height stand on either side of a roadway which is 180 metres wide. The angle of elevation of
the top of the pillars are 600 and 300 at a point on the roadway between the pillars. Find the height of the pillars
and the position of the point.
Two lamp posts are 60 metres apart, and the height of the one is double that of the other. From the middle of the
line joining their feet, an observer finds the angular elevation of their top to be complementary. Find the height of
each lamp.
Two lamp posts are of equal height. A boy measured-the elevation of the top of each lamp-post from the midpoint of the line-segment joining the feet of lamp-post as 300. After walking 15 m towards one of them, he
measured the elevation of its top at the point where he stands as 600. Determine the height of each lamp-post and
the distance between them.
When the suns altitude increases from 300 to 600, the length of the shadow of a tower decreases by 100 metres.
Find the height of the tower.
The angle of elevation of the top of a tower from two points at distances a and b metres from the base and in the
same straight lien with it are and respectively. Prove that the height of the tower is :
From the top of a church spire 96 m high, the angels of depression of two cars on a road, at the same level as the
base of the spire and on the same side of it are and where tan

18.

1
1
and tan Calculate the distance
7
4

between tow cars.


At a point on the level ground, the angle of the elevation of a vertical tower is found to be such that its tangent is

5
3
. On waling 192 m towards the towards, the tangent of the angel is found to be . Find the height of the
12
4
19.

20.

21.

22.

tower.
AB is a straight road leading to C, the foot of a tower, A being at a distance of 120 m from C and B being 75 m
nearer. It the angle of elevation of the tower at B be the double of the angle of elevation of the tower at A, find the
height of the tower.
An aeroplane is observed at the same time by two anti-aircraft batteries distant 6000 m apart to be at elevation of
300 and 450 respectively. Assuming that the aeroplane is traveling directly towards the two batteries, find its
height and its horizontal distance from the nearer battery.
AT.V tower stands vertically on a bank of canal. From a point on the other bank directly opposite the tower the
angle of elevation of the top of the tower is 600. From a point 20 m away from this point on the same bank, the
angle of elevation of the top of the tower is 300. Find the height of the tower and the width of the canal.
A car is traveling on a straight road leading to a tower. From a point at a distance of 500 m from the tower as seen
by the driver is 300. After driving towards the tower for 10 seconds, the angle of elevation of the top of the tower
as seen by the driver is found to be 600. Find the speed of the car.

30

R E

-F O

U N D A T IO N

B Y

MANISH

MATHEMATICS

KUMAR

23.

24.
25.

26.

The height of a hill is 3300 metres. From a point P on the ground the angle of elevation of the top of the hill is
600. A balloon is moving with constant speed vertically upwards from P. After 5 minutes of its movement, a
person sitting in it observes the angle of elevation of the top of the hill as 300. What is the speed of the balloon ?
A man in a boat rowing away from a light-house 100 m high, takes 2 minutes to change the angle of elevation of
the top of the light-house from 600 to 450. Find speed of the boat.
From a point on the ground 40 m away from the foot of tower, the angle of elevation of the top of the tower is 300.
The angle of elevation to the top of a water tank (on the top of the tower) is 450. Find
(i) The height of the tower
(ii) The depth of the tank.
At a point on a level plane, a tower subtends an angle and a man h metres high on its top an angle . Prove that
the height of the tower is :

27.

28.
29.

30.

31.
32.

33.
34.
35.

36.
37.

38.

39.

h tan
tan( ) tan

A vertical tower stands on a horizontal plane and is surmounted by a flagstaff of height 12 metres. At a point on
the plane, the angle of elevation of the bottom of the flagstaff is 450 and of the top of the flagstaff is 600.
Determine the height of the tower:
The angles of elevation of the top and the bottom of a flagstaff fixed on a wall are 450 and 300 to a man standing
on the other end of the road 20 metres wide. Find the height of the flagstaff and the height of the wall.
An aeroplane when flying at a height of 4000 m from the ground passes vertically above another aeroplane at an
instant when the angles of the elevation of the two planes from the same point on the ground are 600 and 450
respectively. Find the vertical distance between the aeroplane at that instant.
An aeroplane when 6000 metres high passes vertically above another aeroplane at an instant when the angles of
the elevation at the same observing point are 600 and 450 respectively. How many metres higher is the one than
the other ?
Two aeroplane are observed to be in a vertical line. The angle of the upper plane is and a that of the lower is
. If the height of the former be H metres, find the height of the latter plane if = 600, = 450, H = 3500 m.
The angle of elevation of a Jet fighter from a point A on the ground is 600. After 10 seconds flight, the angle of
the of elevation changes to 300. If the Jet is flying at a speed of 432 km/hour, find the height at which the jet is
flying.
The angle of elevation of a Jet fighter from a point on the ground is 600. After 15 seconds flight, the angle of the
of elevation changes to 300. If the Jet is flying at height of 1500 3 m, find the speed of the Jet.
From the top of tower 60 metres high, the angle of depression of the top and bottom of a pole are observed to be
450 and 600 respectively. Find the height of the pole and distance of tower from the pole.
From the top of a building 60 metres high, the angle of depression of the top and bottom of a vertical lamp-post
are observed to be 300 and 600 respectively. Find :
(i) The horizontal distance between the building and the lamp-post and
(ii) The difference between the height of the building and the lamp-post.
From the top of a cliff 200 metres high, the angles of depression of the top and bottom of a tower are observed to
be 300 and 600. Find the height of the tower and calculate the distance between them.
A man on the deck of a ship is 12 m above water level. He observes that the angle of elevation, of the top of a cliff
is 450 and the angle of depression of its base is 300. Calculate the distance of the cliff from the ship and the height
of the cliff.
From a window (60 metres high above the ground) of a house in a street the angles of elevation and depression of
the top and the foot of another house on opposite side of street are 600 and 450 respectively. Show that the height
of the opposite house is 60 ( 3 1) metres.
A man on the deck of a ship, 16 m above water level, observes that the angle of elevation and depression
respectively of the top and bottom of a cliff are 600 and 300. Calculate the distance of the cliff from the ship and
the height of the cliff.

31

R E

-F O

U N D A T IO N

B Y

MANISH

MATHEMATICS

KUMAR

40.
41.

The angle of elevation of a cloud from a point 100 m above a lake is 300 and the angle of depression of its
reflection in the lake is 600. Find the height of the cloud.
If the angle of elevation of a cloud from a point h metres above a lake be , and the angle of depression of its

tan tan

tan tan
If the angle of elevation of a cloud from a point h metres above a lake be , and the angle of depression of its

reflection in the lake be , prove that the height of the cloud is : h


42.

reflection in the lake be , prove that the distance (x) of the cloud from the point of observation is :

2h sec
. Find x if = 300, = 450 and h = 250 m.
tan tan

TRIGONOMETRY

EXERCISE-2 (X)-CBSE

ANSWER KEY

VERY SHORT ANSWER TYPE QUESTION :


1.

5
5
2
2.
4.
5.1 6. 450 7. 300 8.
14
3
3

3 9. 2 10. 1 11.

4
2
13.
14. 10 m 15. 300
3
3

SHORT ANSWER TYPE QUESTION :

1.

5
2.
11

p2 q2 q
p q q
2

3.

71
63
31
3
5.
6.
7. 1,
8. 3 9. (i) A = B 450 (ii) A = 180, B = 240
5
65
17
97

6 2
13. 450 14. (i) 200 (ii) 600 (iii) 300
4
10
2
16
83
16. (i)
(ii)
(iii)
(iv) 2 17. (i) , (ii) 3, (iii) 8, (iv) 450, (v) 150, (vi) 150 18. 600 61. 600 62. 40 m
3
8
3
3
63. 173.2 m 64. 17.32 m, = 300 65. 600 66. 600 67. 600 68. 9( 2 1) m , 9 m
10. (i) 200 (ii) 600 12. (i) 2 3 (ii)

LONG ANSWER TYPE QUESTION :


1. 8.66 m 2. 10 m 3. 154 m 4. 69.28 m 5. 45 m 6. 3 m 7. 86.5 m ; 57.67 m, 28.83 m 8. 109.28 m 9. 94.64 m
10. (i) 158.5 m (ii) 173.2 m 11. 42.42 m 12. 135 m from one end, h = 77.94 m
13. 21.21 m, 42.42 m 14. Distance = 45m, height = 12.99 m 15. 86.6 m 17. 288 m 18. 180 m 19. 60 m
20. 3000 ( 3 1) m, 3000 ( 3 1) m, 21. Height = 17.32 m, width = 10 m 22. 120 km/hr 23. 26.4 km/hr
24. 1.269 km/hr 25. (i) 23.1 m (ii) 16.91 m 27. 16.392 m 28. 8.45 m, 11.55 m 29. 1690.66 m, 30. 2536 m
31. 2020.78 m 32. 1039.2 m 33. 720 km/hr 34. h = 25.36 m, x = 34.64 m 35. (i) 34.64 m, (ii) 20 m

1
3

36. Height = 133 m, Distance = 115.46 m 37. Height = 32.784 m, Distance = 20.784 m
39. Height = 48 m, Distance = 27.71 m 40. 200 m 42. 1366 m

32

R E

-F O

U N D A T IO N

B Y

MANISH

MATHEMATICS

KUMAR

EXERCISE 3

1.
2.
3.
4.
5.
6.
7.
8.

(FOR SCHOOL/BOARD EXAMS)

PREVIOUS YEARS BOARD QUESTIONS


SHORT ANSWER TYPE 1
0
0

Without using tables, find the value of 14 sin 30 + 6 cos 60 = 5 tan 450.

[ICSE-2004]

cos ecA
cos ecA
Prove that :

2 2 tan 2 A
cos ecA 1 cos ecA 1
sec . cos ec (90 0 ) tan cot(90 0 ) sin 2 550 sin 2 350
Evaluate :
tan 10 0 tan 20 0 tan 60 0 tan 70 0 tan 80 0
0

[CBSE-Al-2004C]
[CBSE-Al-2004C]

Without using mathematical tables, find the value of x if cos x = cos 60 cos 300 + sin 600 sin 300.
[ICSE-2005]

2 tan 530 cot 80 0


Without using trigonometric tables, evaluate :

cot 37 0 tan 10 0
sin 80 0
Without using trigonometric tables, evaluate :
sin 59 0 sec 310
0
cos10
sin 250
cos 250
Without using tables, evaluate :

sec 650 cos ec650


sin A
(cos ecA cot A)
Prove the
(1 cos A)

[ICSE-2006]
[ICSE-2007]
[ICSE-2008]
[ICSE-2008]

9.

In the fig AD = 4 cm, BD = 3 cm and CB = 12 cm, find cot

10.

11sin 70 0 4
cos 530 cos ec37 0
Without using the trigonometric tables, evaluate the following :

7 cos 20 0 7 7 tan 150 tan 350 tan 550 tan 750

[CBSE-Delhi-2008]

[CBSE-Delhi-2008]
0

11.

Without using the trigonometric tables, evaluate the following :

tan 800 ]

12.
13.

sin 18
3 [tan 10 0 tan 30 0 tan 40 0 tan 50 0
0
cos 72
[CBSE-Delhi-2008]
[CBSE-Al-2008]

If sin = cos , find the value of .


Without using the trigonometric tables, evaluate the following : (sin 2 25 0 sin 2 65 0 ) 3 (tan 50 tan 15 0 tan

30 0 tan 750 tan 850 ) .

[CBSE-Al-2008]

14.

Without using trigonometric tables, evaluate the following : (cos 25 cos 65 ) cos ec sec(90 )
cot tan(90 ) .
[CBSE-Al-2008]

15.

If 7 sin 2 3 cos 2 4 show that tan

16.

If tan A

1
.
3

5
, find the value of (sin A + cos A) sec A.
12

[CBSE-Al-2008]
[CBSE-Foreign-2008]

33

R E

-F O

U N D A T IO N

B Y

MANISH

17.

KUMAR
If sec 4A = cosec (A 200), where 4A is an acute angle, find the value of A.
OR

In a ABC, right angled at C, if tan A


18.
19.
20.
21.
22.

MATHEMATICS
[CBSE-Foreign-2008]

1
, find the value of sin A cos B + cos A sin B.
3

7
, find the value of tan A + cot A.
25
1
If sin = , find the value of [2 cot2 + 2)
3
sin 3 cos 3
sin cos
Simplify :
sin cos
If sec 2 (1 sin ) (1 sin ) k , then find the value of k.
(2 2 sin )(1 sin )
15
, then evaluate
If cot
8
(1 cos )(2 2 cos )
If cos A =

[CBSE-Foreign-2008]
[CBSE-Delhi-2009]
[CBSE-Delhi-2009]
[CBSE-Al-2009]

OR
Find the value of tan 600 geometrically.
23.
24.

[CBSE-Al-2009]

15
and A + B = 900. find the value of cosec B.
[CBSE-Foreign-2009]
If sec A =
7
7 cos 70 0 3
cos 550 cos ec350
Without using trigonometric tables, evaluate :

2 sin 20 0 2 2 tan 50 tan 250 tan 450 tan 850 tan 650
[CBSE-Foreign-2009]
SHORT ANSWER TYPE II

1.
2.

sec 2 54 0 cot 2 36 0
Evaluate :
2 sin 2 380 sec 2 52 0 sin 2 450
2
0
2
0
cos ec 57 tan 33
Prove that following : (tan A tan B ) 2 (1 tan A tan B ) 2 sec 2 A sec 2 B .

[Al-2005]
[Foreign-2005]

OR

sin 150 cos 750 cos 150 sin 750


cos sin(90 ) sin cos(90 )
sec tan 1
cos
.

Prove that:
tan sec 1 1 sin
Evaluate : sec 2 10 0 cot 2 80 0

3.

[Foreign-2005]

OR
Without using trigonometric tables, evaluate the following :

cot(90 ).sin (90 ) cot 40 0

(cos 2 20 0 cos 2 70 0 ) .
sin
tan 450

4.

Without using trigonometric tables, evaluate the following :

5.

sec 2 cot 2 (90 )


(sin 2 40 0 sin 2 50 0 ) .
2
0
2
0
cos ec 67 tan 23
Prove that: (1 tan A) 2 (1 tan A) 2 2 sec 2 A .
sin tan
Prove that:
1 sec .
1 cos

6.

[Delhi-2005C]

[Al-2005C]
[ICSE-2005]
[ICSE-20056]

34

R E

-F O

U N D A T IO N

B Y

MANISH

MATHEMATICS

KUMAR

7.

Prove that:

sin cos sin cos


2 sec 2
.

sin cos sin cos tan 2 1

[ICSE-20056]

OR

sec (90 ) cot 2 2 cos 2 60 0 tan 2 28 0 tan 2 62 0


.

Without using trigonometric tables :


2(sin 2 25 0 sin 2 650 )
3(sec 2 430 cot 2 47 0 )
1
1
1
1
Prove that :

cos ec cot sin sin cos ec cot


2

8.

[Al-2006]

OR

cos ec (90 ) tan 2 2 tan 2 30 0 sec 2 52 0 sin 2 380


.

4(cos 2 480 cos 2 42 0 )


cos ec 2 70 0 tan 2 20 0
sin 2 sin 2 (90 ) 3 cot 2 30 0 sin 2 54 0 sec 2 36 0
Without using trigonometric tables :
.

3(sec 2 610 cot 2 29 0 )


2(cos ec 2 650 tan 2 250 )
2

Without using trigonometric tables :

9.
10.

[Foreign-2006]

Without using trigonometric tables evaluate the following :

cos 2 20 0 cos 2 70 0
2 cos ec 2 580 2 cot 580 tan 32 0 4 tan 130 tan 37 0 tan 450 tan 530 tan 77 0
2
0
2
0
sec 50 cot 40
OR
Prove that :
11.

sec 1
sec 1

2 cos ec
sec 1
sec 1

[Delhi-2006C]

Without using trigonometric tables evaluate the following :

sec 39 0
2

tan 17 0 tan 38 0 tan 60 0 tan 52 0 tan 730 3(sin 2 310 sin 2 59 0 )


0
cos ec51
3
3 cos 55 0
4(cos 70 0. cos ec 20 0 )
(ii)

7 sin 350 7(tan 5 0. tan 25 0. tan 45 0. tan 65 0. tan 85)


cot 54 0
0
0
0
0
0
(iii) tan 7 . tan 23 . tan 60 . tan 67 . tan 83
sin 20 0.sec 70 0 2
0
tan 36
sin A 1 1 cos A
Prove that :

sin A 1 1 cos A
cot A cos A cos ecA 1
Prove that :

cot A cos A cos ecA 1


(i)

12.
13.

14.

15.

OR
Prove that : (1 + cot A cosec A) (1 + tan A + sec A) = 2
Prove that : (sin cos ec ) 2 (cos sec ) 2 7 tan 2 cot 2
OR
Prove that sin (1 + tan ) + cos (1 + cot ) = sec + cosec
Prove that : (1 cot A tan A)(sin A cos A) sin A tan A cot A cos tA .
OR

[Al-2006C]
[Delhi-2007]
[Al-2007]
[ICSE-2007]

[CBSE (Delhi)-2008]

[CBSE - Al-2008]
[CBSE-foreign-2008]

cos 58 0
cos 38 0 cos ec52 0
.
3

0
0
0
0
sin 32
tan 15 tan 60 tan 75

Without using trigonometric tables evaluate the following : 2

35

R E

-F O

U N D A T IO N

B Y

MANISH

16.

MATHEMATICS

KUMAR
Find the value of sin 300 geometrically.
OR
Without using trigonometric tables, evaluate :

cos 580 sin 22


cos 380 cos ec52 0

sin 32 0 cos 680 tan 180 tan 350 tan 60 0 tan 72 0 tan 550
[CBSE-Delhi-2009]

17.
18.

1.

2
2
5
cos ec 2 580 cot 580 tan 32 0 tan 130 tan 37 0 tan 450 tan 530 tan 77 0 . [CBSE-Al-2009]
3
3
3
2
4
sin 2 sin
Prove that : sec 2
1
[CBSE-foreign-2009]
2 cos 4 cos 2
LONG ANSWER TYPE
Evaluate :

On a horizontal plane there is a vertical tower with a pole on the top of the tower. At a point 9 metres away from
the foot of the tower the angle of elevation of the top and bottom of the flag pole are 600 and 300 respectively.
Find the height of the tower and flag pole mounted on it.
OR
From a building 60 metres high the angles of depression of the top and bottom of lamp-post are 300 and 600
respectively. Find the distance between lamp-post and building. Also find the difference of height between
building and lamp-post.
[Delhi-2008]

2.

From the top of a cliff 92 cm high, the angle of depression of a buoy is 200. calculate to the nearest metre, the
distance of the bucy from the foot of the cliff.
[ICSE-2005]

3.

The shadow of a vertical tower AB on level ground is increased by 10 m, when the altitude of the sun changes
from 450 to 300. Find the height of the tower and give your answer correct to

4.

1
of a metre.
10

[ICSE-2006]

The angle of depression of the top and the bottom of a building 50 metres high as observed from the top of a
tower are 300 and 600 respectively. Find the height of the tower and also the horizontal distance between the
building and the tower.
[Delhi-2006]
OR
The angle of elevation of the top of a tower as observed from a point on the ground is ' ' and on moving a
metres towards the tower, the angle of elevation is ' ' Prove that the height of the tower is

5.

6.

7.

a tan tan
.
tan tan

A man on the top of a vertical tower observes a car moving at a uniform speed coming directly tower it. If it takes
12 minutes for the angle of depression to change from 300 to 450 how soon after this, will the car reach the tower?
[Al-2006C]
A boy standing on a horizontal plane finds at a distance of 100 m from him at an elevation of 300. A girl standing
on the roof of 20 metre high building, finds the angle of elevation of the same bird to be 450. Both the boy and the
girl are on opposite sides of the bird. Find distance of bird from the girl.
[Delhi-2007]
Statue 1.46 m tall, stands on the top of the pedestal . From a point on the ground, the angle of elevation of the top
of the statue is 600 and from the same point, the angle of elevation of the top of the pedestal is 450. Find the height
of the pedestal. (use

3 1.73 )

[CBSE-Delhi-2008]

36

R E

-F O

U N D A T IO N

B Y

MANISH

MATHEMATICS

KUMAR

A person standing on the bank of a river observes that the angle of elevation of the top of a tree standing on the
opposite bank is 600. When he moves 40 m away from the bank, he finds the angle of elevation to be 300. Find the

8.

height of the tree and the width of the river. (use

3 = 1.732)

[CBSE-Delhi-2008]

The angle of elevation of a jet fighter from a point A on the ground is 600. After a fight of 15 seconds, the angle of
elevation changes to 300. If the jet is flying at a speed of 720 km/hour, find the constant height at which the jet is

9.

flying. (use

3 = 1.732)

[CBSE-Al-2008]

10.

The angle of elevation of an aeroplane from a point A on the ground is 600. After a flight of 30 seconds, the angel
of elevation changed to 300. If the plane is flying at a constant height of 3600 3 m, find the speed in km/hour of
the plane.
[CBSE-foreign-2008]

11.

A straight highway leads to the foot of a tower. A man standing at the top of the tower observes a car at an angle
of depression of 300, which is approaching the foot of the tower with a uniform speed. Six seconds later the angle
of depression of the car is found to be 600. Find the time taken by the car to reach the foot of the tower from this
point.
[CBSE-Delhi-2009]

12.

An aeroplane when flying at a height of 3125 m from the ground passes vertically below another plane at an
instant when the angels of elevation of the two planes from the same point on the ground are 300 and 600
respectively. Find the distance between the planes at that instant
[CBSE-Al-2009]

13.

A man is standing on the deck of a ship which is 25m above water level. He observes the angle of elevation of the
top of a lighthouse as 600 and the angle of depression of the base of the light house as 45+0. Calculate the height
of the lighthouse.
[CBSE-foreign-2009]

TRIGONOMETRY

ANSWER KEY

EXERCISE-2 (X)-CBSE

SHORT ANSWER TYPE QUESTION-I

2
12
17
625
4. 300 5. 1 6. 2 7. 1 9.
10. 1 11. 2 12. 450 13. 2 14. 2 16.
17. 220 or 1 18.
5
12
168
3
625
15
or 3 23.
24. 5
19. 18 20. 1 21. 1 22.
64
7
1 5 3.

SHORT ANSWER TYPE QUESTION-II

1
5
2
25
5
2. or 2 3. or 1 4. 2 7. or
8. or
9.
10. 1 11. (i) 0 (ii)
(iii)

2
3
6
7
12
2 3 1
1
16.
or
17. 1
2
3
1.

3 1 15. or 1

LONG ANSWER TYPE QUESTION-II


1. 15.588 m, 5.196 m or 34.64 m and 20 m 2. 253 m 3. 13.66 m 4. 75 m and 43.3 m 5. 16 minutes 23 seconds
6. 30 2 7. 2 m 8. 34.64 m and 20 m 9. 2598 m 10. 864 km/hr 11. 3 seconds 12. 2083.33 m 13. 68.25 m

37

R E

-F O

U N D A T IO N

B Y

MANISH

MATHEMATICS

KUMAR

EXERCISE 4

(FOR OLYMPIADS)

CHOOSE THE CORRECT ONE

ax sin by cos
ax
by

a 2 b 2 and

0 then (ax)2/3 + (by)2/3 is equal to :


2
2
cos
sin
cos sin

1.

If

2.

(A) (a2 b2)2/3


(B) (a2 + b2)2/3
(C) (a b)2/3
(D) None of these
The sides of a right angled triangle form a geometric progression, find the cosines of the acute angles. (If a, b, c are in G.P. b2 = ac) :

5 1
5 1
5 1
5 1
and
(B)
and
2
2
2
2
5 1
5 1
(C)
and
(D) None of these
2
2
1 cos sin
2 sin
, then
is equal to :
If y
1 cos sin
1 sin
1
(B) 1 y
(C)
(D) None of these
(A) 1+ y
y
0
0
(A)

3.

4.

cot 36 cot 72 is equal to :


(A)

5.

1
5

(B)
2

(C) 1
2

(D) None of these


2

The value of cos 15 cos 30 + cos 45 cos 60 + cos 75 is :


(A) 2

6.

1
50

If

(B) 0

(C)

x sin 2 cos and y cos 2 sin , then :


2

(A) ( x y )

2/3

( xy )

2 2/3

1
4

x2
(B)
y

(D)
2/3

y2

x

2/3

7.

(C) x2 + y2 = x2y2
(D) None of these
If x sec tan and y cos ec cot , then xy 1 is equal to :
(B) x y
(C) 2x + y
(A) x + y

8.

If 5 sin 3 , then
(A)

9.

1
4

11.

12.

(B) 4

The value of the expression 1


(A) cos y

10.

sec tan
is equal to :
sec tan

(B) 1

(C) 2

1
2

(D) y x

(D) None of these

sin 2 y 1 cos y
sin y
is equal to :

1 cos y
sin y
1 cos y
(C) 0

(D) sin y

1
If sec x
, x R, x 0 , then the value of sec tan is :
4x
1
1
(A) 2x
(B)
(C) 2x or
(D) None of these
2x
2x
p sin q cos
p
is :
If tan , then the value of
q
p sin q cos
p2 q2
p2 q2
(B) 2
(C) 0
(D) None of these
(A) 2
p q2
p q2
2
2 2
If m = tan sin and n tan sin , then (m n ) is equal to :
38

R E

-F O

U N D A T IO N

B Y

MANISH

MATHEMATICS

KUMAR

13.
14.
15.
16.
17.

(A) mn
(B) 4 mn
(C) 16 mn
If x cos b sin and y a sin cos then a2 + b2 is equal to :
(A) x2 y2
(B) x2 + y2
(C) (x+ y)
If cos

(B) 0
(C) 2
(D) 1
(A) 2
ABC is a triangle, right angled at A. If the length of hypotenuse is 2 2 times the length of perpendicular from A
on the hypotenuse, the other angles of the triangle are :
(A) 22.50, 67.50
(B) 300, 600
(C) 450, 450
(D) None of these
3
3
If sin A + cos A = m and sin A + cos A = n, then :
(B) m3 3m + 2n =
(A) m3 + 3m + 2n = 0
3
(D) m3 3m + n = 0
(C) n 3n + 2m = 0
If sin 2 3 cos 2 0 , then cos3 sec3 is equal to :
(B) 9

20.
21.

3 2
(a 1) 2
4

If 0 x

or

25.

26.

1
4

3 2
(a 1) 2
4

(C)

3 4(a 2 1) 2
4
0

(D)

3 3(a 2 1) 2
4

1
, then the values of A and B lying between 00 and 900 are respectively:
20
0
0
0

(B) 60 and 30

(C) 45 15

(D) None of these

and 81sin 2 x 81cos 2 x 30 , then x is equal to :


(B)

or 0

(C)

or

(D) None of these

4
2
4
6
If m 2 m '2 2mn' cos 1, n 2 n '2 2nn' cos 1 , and mn m' n'(mn' m' n) cos 0 , then m2 + n2 is
3

equal to :
(A) sin 2
24.

(B) 1

If sin( A B ) cos( A B )

(A)
23.

(D)

The quadratic equation whose roots are sin 180 and cos 36 is :
(A) 4 x 2 2 5 x 1 0 (B) 4 x 2 2 5 x 1 0 (C) x 2 2 5 x 1 0 (D) 4 x 2 2 5 x 1 0
If cos sec 2 , then the value of cos 2 sec 2 is :
(A) 1
(B) 2
(C) 4
(D) None of these

(A) 300 and 600


22.

(C) 4

If sin cos a , then sin 6 cos 6 is equal to :


(A) 1

19.

(D) None of these

x
y
y
x
y
sin 1 0 and sin cos 1 0 then 2 2 is equal to :
b
a
b
a
b

(A) 18
18.

(D) 4 mn

(B) cos ec 2

(C) cos 2

cos A
sin A
p and
q , then tan A is equal to :
sin B
cos
p q2 1
q2 1
p q2 1
(B)
(C)
(A)

q 1 p2
1 p2
q 1 p2
T T5
If Tn sin n cos n , then 3
is equal to :
T1
T T7
T T5
T T6
(A) 5
(B) 3
(C) 9
T7
T4
T3

(D) None of these

If

The number of values of which lie between 0 and


(A) 1

(B) 2

(C) 3

(D) None of these

(D)

T6 T9
T4

and satisfy the equation sin 4 2 sin 2 1 0 is :


(D) None of these

39

R E

-F O

U N D A T IO N

B Y

MANISH

MATHEMATICS

KUMAR

27.

The greatest angle of a cyclic quadrilateral is 3 times least. The circular measure of the least angle is :
(A) 600

28.

(B)

30.

31.
32.
33.

If 0 < x <

3 1
m
3 1

(B)

(B) 30 (3 3 ) m

a2
3

3 1

3 1 60 m

(C)

(D) None of these

(C) 30 (3 3 ) m

(D) 30 ( 3 1) m

(C) 1

(D) None of these

(B) 1

, then the largest angle of a triangle whose sides are 1, sin x, cos x is :
(B)

2
a2
bc

(C)

(D) x

(B)

c2
ab

(C)

b2
x
ac

(D) a + b

A rectangle with an area of 9 square metre is inscribed in a triangle ABC having AB = 8 m, BC = 6 m and
ABC 90 0 . The dimensions of the rectangle (in metres) are :
(A) 2,

3
9
or 6,
2
2

(B) 1 , 9 or 3, 3

(C) 2, 4.5

(D) 4, 2.25

From the top of a light house, the angles of depression of two stations on opposite sides of it at distance a apart
are and . The height of the light house is :
(A)

40.

(D)

ABC is right angled at C, then tan A + tan B =


(A)

39.

a2
6

sin cos 3 sin cos is equal to :


6

(A)

38.

(C)

The angles of elevation of the top of a tower as observed from the bottom and top of a building of height 60 m are
600 and 450 respectively. The distance of the base of the tower from the base of the building is :

(A) 0

37.

(D) None of these

(A) cos 2 cos 2


(B) tan 2 tan 2
(C) tan 2 tan 2
(D) sec 2 sec 2
From the top of a light house, 60 m high with its base at the sea level, the angle of depression of a boat is 150. The
distance of the boat from the foot of the light house is :

(A) 30 ( 3 1) m

36.

If sin x + sin2x = 1, then the value of cos12x + 3cos8x + cos6x + 2cos4x + cos2x 2 is equal to :
(A) 0
(B) 1
(C) 2
(D) sin2x
The angles of elevation of the top of a TV tower from three points A, B and C in a straight line (in the horizontal
plane) through the foot of tower are , 2 and 3 respectively. If AB = a, the height of tower is :
(A) a tan
(B) a sin
(C) a sin 2
(D) a sin 3
2
2
2
2
2
2
2
The expression cosec A cot A sec A tan A (cot A tan A) (sec A cosec2A 1) is equal to :
(A) 0
(B) 1
(C) 1
(D) None of these
2
2
(1 tan tan ) (tan tan ) is equal to :

3 1

3 1 60 m

35.

(B) 3a2

(A)
34.

(C)

A circle is inscribed in an equilateral triangle of sides a, the area


of any square inscribed in the circle is :
(A) 6a2

29.

a
cot cot

(B)

a
cot cot

(C)

a cot cot
cot cot

The value of the expression tan 10 tan 20 tan 30tan 890 is equal to :
(A) 0
(B) Not defined
(C) 1

(D)

a tan tan
cot cot

(D)

40

R E

-F O

U N D A T IO N

B Y

MANISH

MATHEMATICS

KUMAR

41.
42.
43.

If sin 1 sin 2 sin 3 3 then cos 1 cos 2 cos 3 is equal to :


(A) 3
(B) 2
(C) 1
(D) 0
If sinx + sin2x = 1, then cos8x + 2cos6x + cos4x is equal to :
(A) 0
(B) 1
(C) 2
(D) 1
Which of the following is not possible ?

5
1 t 2
(B) cos
, t 0 (C) tan 100
1 t 2
7
cot 2 sin cos (0 90 0 ) if equals :

(A) sin
44.
45.

46.


:
n n


.
n n

49.
50.


:
n n

(B) cos

If tan sec 3 , 0
(A)

48.

5
2

(A) 450 and 900


(B) 450 and 600
(C) 450 only
(D) 900 only
In a triangle ABC right angled at C, tan A and tan B satisfy the equation :
(A) abx2 (a2 + b2) x ab = 0
(B) abx2 c2x + ab = 0
2 2
2
(C) c x abx + c = 0
(D) ax2 bx + a = 0
The area of the circle and the area of a regular polygon of n sides and of perimeter equal to that of the circle are in
the ratio of :
(A) tan

47.

(D) sec

(B)


:
n n

(C) sin

(D) cot

, then is equal to :
(C)

(D) None of these

A tower subtends an angle at a point A in the plane of its base and the angle of depression of the foot of the
tower at a height b just above A is B. Then the height of the tower is :
(B) b cot tan
(C) b tan tan
(D) b cot cot
(A) b tan cot
If sinx + sin2 x = 1, then cos2x + cos4x is equal to :
(A) 1
(B) 1
(C) 2
(D) 0
The angle of elevation of a tower from a point A due south of it is x and from a point b due to east of A is y. if
AB = , the height h of the tower is :
(A)

(B)

cot y cot x
2

tan y tan x
2

OBJECTIVE

(C) cot 2 y cot 2 x (D) tan 2 y tan 2 x

EXERCISE -4

ANSWER KEY

Que.
Ans.

1
A

2
C

3
D

4
B

5
D

6
B

7
D

8
B

9
A

10
C

11
A

12
C

13
B

14
A

15
A

Que.
Ans.
Que.
Ans.
Que.
Ans.

16
B
31
A
46
A

17
A
32
D
47
B

18
B
33
A
48
A

19
D
34
D
49
A

20
C
35
B
50
A

21
C
36
A

22
A
37
A

23
B
38
A

24
A
39
B

25
A
40
C

26
D
41
D

27
B
42
D

28
C
43
B

29
D
44
A

30
C
45
B

41

R E

-F O

U N D A T IO N

B Y

MANISH

MATHEMATICS

KUMAR

COMPETITION WINDOW
LAW OF SINES
We use the sine rule for non-right angled triangles to find the lengths and angles. In trigonometry, the law of sines
(also known as the sines law, sine formula, or sine rule) is an equation relating the lengths of the sides of an
arbitrary triangle to the sines of its angle. According to the law.

a
b
c

sin A sin B sin C


Where a, b and c are the lengths of the sides of a triangle, and A, B and C are the opposite angles. To use the sine
rule, choose an appropriate pair, depending on what you know in the triangle
e.g.,

b
c
a
b
a
c

or

or

sin A sin B
sin a sin C sin B sin C

If you are finding an angle, you can invert the formulae.


e.g.,
E.g.

sin A sin B
sin a sin C sin B sin C

or

or

b
c
a
b
a
c

Find the length of PQ in triangle PQR.


Use the sine rule

p
q
r

sin P sin Q sin R


q
r
Use :

sin Q sin R
r
165

0
sin 84
sin 230
165 sin 230
r
64.8 metres
sin 84 0
1.
2.

TRY OUT THE FOLLOWING


Find the length of BC in triangle ABC, if A 780 , C 180 and AB = 26 cm.

[Ans : 82.3 cm]

Find the length of AC in triangle ABC, if B 86 , A 74 and AB = 35 cm.


RELATION TO THE CIRCUM RADIUS (R)
0

a
b
c

2R
sin A sin B sin C

[Ans : 102.1 m]

(i)

AREA OF A TRIANGLE
For any triangle ABC, the area is given by

B
1
1
1
bc sin A ca sin B ab sin C
2
2
2

C
(ii)

42

R E

-F O

U N D A T IO N

B Y

MANISH

MATHEMATICS

KUMAR

From (i)

sin C =

c
2R

(iii)

1
ab sin C
2
abc
From (iii) and (iv) R
4
From (ii)

(iv)

LAW OF COSINES
In any triangle ABC,

b2 c2 a2
cos A
2bc
2
a c2 b2
cos B
2ac
2
a b2 c2
cos C
2ab

A
c
B

b
C

To use the cosine rule, you need to know either two sides and the included angle or all three sides.
e.g., Find the length of AC in ABC
use :

a2 c2 b2
2ac
(4) 2 (4 2 ) 2 AC 2
cos 450
2 4 4 2

cos B

32 = 48 AC2
AC = 4 cm

A
4 cm

4 2 cm

43

R E

-F O

U N D A T IO N

B Y

MANISH

MATHEMATICS

KUMAR

EXERCISE 5

(FOR IIT-JEE/AIEEE)

CHOOSE THE CORRECT ONE


Based On Sine Rule (Q.No. 1-20)
1.

In ABC. AB = 30 cm and C = 450. The length of the radius of circumcircle of ABC is :


(B) 5 2 cm
(C) 15 3 cm
(D) 5 3 cm
(A) 15 2 cm

2.

The radius of the circumcircle of ABC is

3.

(A) 300
(B) 600
(C) 90
(D) 45
In ABC, A : B = 1 : 3 : 8. If AB = 10 cm, the length of AC is : [Use : is (180 0 ) = sin ]
(A)

4.

5.
6.

10 6
cm
3

(B)

10 3
cm
3

2 3
cm. If BC = 2 cm, the size of angle A is :
3
0
0

(C)

The measure of angle x in the triangle below is :

10 3
cm
6

(B) 57.010
(C) 590
(D) None of these
(A) 540
In a circle of radius 7 cm, the are AB subtends an angle of 1200 at the centre. The length of chord
AB is :
(B) 3 2 cm
(C) 5 3 cm
(D) 2 3 cm
(A) 7 3 cm
In a triangle ABC, a = 6, b = 12 and B = 600. The value of sin A is ;
(A)

3
cm
4

(B)

1
cm
3

(C)

1
cm
2

2
, then B is equal to :
3
0

7.

In ABC. a = 2, b = 3 and sin A

8.

(A) 300
(B) 600
(C) 90
In ABC. a = 4, c = 12 and C = 600, then the value of sin A is :
(A)

9.
10.

1
2 3

cm

(B)

1
cm
2 3

(C)

10 sin 2a
sin 3a

(C)

2
cm
3

(D) None of these

(D) 1200
(D)

3
2

In an isosceles triangle ABC, the base AB = 12 cm and the angle at the top is 300. D is a point on the side BC such
that CAD : DAB = 1 : 4. The length of the radius of circumcircle of ABC is :
(B) 5 2 cm
(C) 6 2 cm
(D) 10 2 cm
(A) 3 2 cm
The base of an isosceles triangle is 10 cm, and the angle at the base is 2a. the length of the angle bisector of one of
the base angles is : [Use : sin(180 0 ) sin ]
(A) 10 sin 2a cos 2a

11.

(D) None of these

(B)

10 sin 3a
sin 2a

(D) 1 0sin 4a

In the circumference with radius 50 cm is inscribed a quadrilateral. Two of its angles are 450 and 1200. The length
of diagonals is :
(B) 10 2 cm; 10 3 cm
(A) 25 2 cm; 25 3 cm
(C) 50 2 cm; 50 3 cm

(D) None of these

44

R E

-F O

U N D A T IO N

B Y

MANISH

MATHEMATICS

KUMAR

12.

In ABC, A = 450, B = 300. M is a point on the side AB. The radius of the circumcircle of AMC is R.
The radius of the circumcircle of MBC is :
(A) 2R cm

13.
14.
15.
16.
17.

(B) R 2 cm

(A) 5 : 1
(B) ( 5 1) : 1
(C) 1 : ( 5 1)
(D) None of these
In a ABC, if a sin A = b sin B, then the triangle is :
(A) Right angled
(B) Equilateral
(C) Right angled isosceles
(D) Isosceles
Points D, E are taken on the side BC of a triangle ABC such that BD = DE = EC. If BAD = x, DAE = y,

sin( x y ) sin( y z )
is equal to :
sin x sin z

(B) 2

(C) 4b

(D) None of these

In a triangle ABC, A = 45 , B = 75 , then a 2 c is equal to :


0

(A) 2b

20.

(D) None of these

(A) 2 + 3 : 1
(B) 2 + 3 : 2 3
(C) 3 1 : 3 + 1
(D) None of these
The perimeter of an acute angled triangle ABC is 6 times the arithmetic mean of the sines of its angles. If the side
b is 2, the angle B is :
(A) 300
(B) 600
(C) 900
(D) None of these
If the angles of a triangle be in the ratio 1 : 4 : 5, then the ratio of its greatest side to the smallest side is :

(A) 1

19.

R
cm
2

The angle of a triangle are as 5 : 5 : 2, the ratio of the greatest side to the least side is :

EAC = z, then the value of


18.

(C)

(B) b

(C) 4b

(D)

b
2

A hiker starts her journey at point A. She notices a farm house at point C and works out its bearing is at 1380. She
then walks for 5 kilometres and stops at point B. At point B the hiker looks again at the farm house and calculates
its bearing now to be 2000. The distance AC and BC respectively are :

(A) 3.28 km, 6.55 km


(B) 2.66 km, 5.83 km
(C) 2.83 km, 5.66 km
(D) None of these
The angles of a triangle are in the ratio 4 : 1 : 1, then the ratio of the largest side to the perimeter is (Use : sin
(180 0 ) sin )
(A) 1 : (1 3 )

(B) 2 : 3

(C)

3 : (2 3 )

(D) 1 : (2 3 )

Based On Cosine Rule (Q. No. 21-37)


21.

In ABC, AB = 5 cm, AC = 6 cm, A = 600. The length of the side BC is :

22.

(A) 31 cm
(B) 29 cm
(C) 31 cm
Which of the following options contains the sides of a right angled triangle ?
(A) 13, 14, 15
(B) 12, 35, 37
(C) 13, 15, 24

(D) None of these

The size of C of ABC, if a = 2 3 cm, b = 3 cm, c =


(A) 900
(B) 600
(C) 300

(D) None of these

23.

(D) 29 cm

3 cm is :
45

R E

-F O

U N D A T IO N

B Y

MANISH

MATHEMATICS

KUMAR

24.

The size of C of ABC, if a = 11 cm, b = 60 cm, c = 61 cm is :


(A) 900
(B) 600
(C) 300

25.

In ABC we have AC = 3 cm, BC =

26.

(A) 3 cm
(B) 3 3 cm
The length of a diagonal of a rectangle is 32 cm, and the angle between the diagonals is 1350. The length of the
sides of rectangle are :

5 cm, A = 45 . The length of the side AB is :


(C) 2 cm or 2 2 cm
(D) 3 cm or 3 3 cm

(A) 4 3 3 cm and 4 3 3 cm
(C) 4 cm and 16 cm
27.

28.

29.

(D) None of these

(B) 16 2 2 cm and 16 2 2 cm
(D) None of these

The incentre of a right angled triangle is at distance 5 and


length of the hypotenuse is :
(A) 5 cm
(B) 10 cm
(C) 15 cm

10 from the two ends of the hypotenuse. The


(D) 7.5 cm

The incentre of ABC is at distance 7 and 3 3 from the point A and B. If the angle at point C is 1200, the
length of the side AB is :
(B) 129 cm
(C) 119 cm
(D) None of these
(A) 139 cm
Calculate the length y of the side in the triangle below :

A
y

5
420

30.

31.
32.

33.

(A) 5.25
(B) 4
(C) 6.25
(D) None of these
A ship sails from harbour and travels 25 km on a bearing of 300 before reaching a marker bouy. At this point the
ship turns and follows a course on a bearing of 900 and travels for 32 km until it reaches an island. On the return
journey, the ship is able to take the most direct route back to the harbour. The total distance triavelled by the ship
is :
(A) 105 km
(B) 95 km
(C) 112 km
(D) 130 km
If the angles of a triangle ABC are in AP, then :
(A) c2 = a2 + b2 + ab
(B) a2 + c2 ac = b2
2
2
2
(C) c = a + b
(D) None of these
If a = 4, b = 3 and A = 600, then c is a root of the equation :
(A) x2 3x 7
(B) x2 + 3x + 7 = 0
2
(C) x 3x + 7
(D) x2 + 3x 7 = 0
If p1, p2, p3 are the altitudes of a triangle from the vertices A, B C and , the area of the triangle,
=

34.

1
1
1

p1 p2 p3

ab(1 k )
, then k is equal to :
(a b c)

(A) cos C

(B) cos A

(C) cos B

(D) None of these

(A) a2 + b2 c2

(B) c2 + a2 b2

(C) b2 c2 a2

(D) c2 a2 b2

A B C
In a ABC, 2ac sin
is equal to :
2

46

R E

-F O

U N D A T IO N

B Y

MANISH

MATHEMATICS

KUMAR

35.

In a triangle the length of two larger sides are 10 and 9 respectively. If the angles are in A. P., then the third side
can be :
[DCE-2001]
(A) 5 6

36.

(B) 5 6

In a ABC if b = 20, c = 21 and sin A =


(A) 12

37.

(C) 3 3

In a ABC,
(A)

5
7

3
, then a =
5

(B) 13

(D) 5
[EAMCET-2003]

(C) 14

bc ca ab
, then cos C =

11
12
13
7
16
(B)
(C)
5
17

(D) 15
[Karnataka-CET-2003]
(D)

17
36

Mixed Applications of Sine & Cosine Rule (Q.No. 38-41)


38.

The sides of a triangle are 3 1 and


is :
(A) 300
(B) 450

3 1 and the included angle is 600. The difference of the remaining angles
(C) 600

(D) 900

39.

If two sides of a triangle and the included angle are given by a = (1 3 ) cm, b = 2 cm, c = 600, the other two

40.

angles are :
(A) 900, 300
(B) 750, 450
In the previous Q., the third side is ;

41.

(B) 6 cm
(A) 6 cm
2
2
2
If b + c = 3a , then cot B + cot C cot A =
(A) 1

(B)

ab
4

(C) 600, 600

(D) None of these

(C) 9 cm

(D) None of these

(C) 0

(D)

ac
4

Based On Area Of Triangle (Q.No. 42-46)


42.

In a triangle ABC, B = 450, a = 2( 3 1 ) and area of ABC = 6 + 2 3 square units, then the side b is equal to
(A)

3 1
2

(B) 4

(C)

2 ( 3 1)

(D) None of these

(a b c)(c b a)(c a b)(a b c)


is equal to :
4b 2 c 2

43.

In any ABC, the expression

44.

(A) cos2A
(B) sin2A
(C) 1 cosA
(D) 1 + cos A
In any ABC, the expression (a + b +c) (a + b c) (b + c a) (c + a b) is equal to :
(A) 16

45.

47.

(C) 4

(D) None of these

bx cy az

=
c
a b
2(a 2 b 2 c 2 )
(D)
R

If x,y,z are perpendiculars drawn from the vertices of a triangle having sides a, b and c, then

a2 b2 c2
a2 b2 c2
(C)
R
4R
In an equilateral triangle of each side 2 3 cm, the radius of the circum circle is :

(A)
46.

(B) 4 2

a2 b2 c2
2R

(B)

(A) 2 cm
(B) 1 cm
(C) 3 cm
(D) 2 3 cm
A pole stands vertically inside a triangular partk ABC. If the angle of elevation of the top of the pole from each
[IIT-2001]
corner of the park is same, then in ABC, the foot of the pole is at the :
(A) Centroid
(B) Circumcentre
(C) Incentre
(D) Orthocentre

47

R E

-F O

U N D A T IO N

B Y

MANISH

MATHEMATICS

KUMAR

48.

A man from the top of a 100 m high tower sees a car moving towards the tower at an angle of depression of 300.
After some time, the angle of depression becomes 600. the distance (in metres) traveled by the car during this time
is :

[IIT-Screening-2001]

(A) 100
49.

(B)

(C)

100 3
3

(D) 200 3

The value of k for which (cosx + sinx)2 + k in sin x cos x 1 = 0 is an identity is : [Kerala Engineering-2001]
(A) 1

50.

200 3
3

(B) 2

(C) 0

(D) 1

Which of the following pieces of does not uniquely determine an acute angled triangle ABC (R beign the radius
of the circumcircle)?
(A) a, sinA, sinB

(B) a, b, c

51.

The value of

(A) a, sinA, sinB


52.

cos 2
(A)

53.

12

(C) a, sinB, R

(D) a, sinA, R.

1 tan 15

1 tan 2 150
2

cos 2

[II Screening-2002]

(B) a, b, c

cos 2

2
3 3

(C) a, sinB, R

(D) a, sinA, R.

5
is equal to :
12

(B)

2
3

[Karnataka-CET-2002]
(C)

3 3
2

(D)

If tanA + cotA = 4, then tan4A + cot4A is equal to :

[Kerala Engineering-2002]

(A) 110

(D) 194

(B) 191

(C) 80

54.

If tan sec e , then cos equals :

55.

2
e x ex
e x e x
(B) x
(C)
e ex
2
2
2
In a ABC, if a2 + b2 + c2 ab bc ca = 0, then sin A + sin2B + sin2C =

[AMU-2002]

(A)

(A)
56.

4
9

(B)

9
4

(C) 3 3

In a triangle ABC, medians AD and BE are drawn. If AD = 4, DAB

(A)

64
3

(D)

e x e x
e x ex

[Karnataka-CET-2003]
(D) 1

triangle ABC is :

57.

2
3

and ABE

, then the area of the

[AIEEE-2003]
(B)

8
3

(C)

32
3

(D)

32
3 3

3
5

3
4

The upper the portion of a vertical pole subtends an angle tan1 at a point in the horizontal plane through
its foot and at a distance 40 m from the foot. A possible height of the vertical pole is :
[Hint : Use the formula tan( )
(A) 60 m

(B) 20 m

tan tan
]
1 tan tan

(C) 40 m

[AIEEE-2003]
(D) 80 m

48

R E

-F O

U N D A T IO N

B Y

MANISH

MATHEMATICS

KUMAR

58.

1
1
. cos , then the value of lies in : [IIT-Screening-2004]
2
3

2 5
5
,
(B) ,
(C)
(D)
,

3 3
3 6
6

If and are acute angles, sin


,
3 2

(A)
59.

The sides of a triangle are in the ratio 1 :

3 : 2, the angles of the triangle are in the ratio :

(A) 1 : 3 : 5

(B) 2 : 3 : 4

(C) 3 : 2 : 1

(D) 1 : 2 : 3

[IIT-Screening-2004]

60.

A person standing on the bank of a river observes that the angle of elevation of the top of a tree on the opposite
bank of the river is 600 and when he retires 40 metres away from the tree the angle of elevation becomes 300. The
breadth of the river is :
[AIEEE-2004]
(A) 20 m
(B) 30 m
(C) 40 m
(D) 60 m

61.

If the roots of the quadratic equation x2 + px + q = 0 are tan 300 and tan 150, then the value of 2 + q p is
(A) 1
(B) 2
[AIEEE-2006]
(C) 3
(D) 0

62.

A tower stands at the centre of a circular park. A and B are two points on the boundary of the park such that AB
(= a) subtends an angle of 600 at foot of the tower, and the angle of elevation of the top of the tower from A or B
is 300. The height of the tower is :
[AIEEE-2007]

2a
3

(A)
63.

(B) 2a 3

a
3

(C)

(D) a 3

AB is a vertical pole with B at the ground level and A at the top. A man finds that the angles of elevation of the
point A from a certain point C on the ground is 600. He moves away from the pole along the line BC to a point D
such that CD = 7 m. From D the angle of elevation of the point A is 450. then the height of the pole is :
(A)

7 3
m
2( 3 1)

(B)

7 3
( 3 1) m
2

(C)

7 3
( 3 1) m
2

(D)

7 3
m
2( 3 1)

OBJECTIVE
Que.
Ans.
Que.
Ans.
Que.
Ans.
Que.
Ans.
Que.
Ans.

1
A
16
D
31
B
46
A
61
C

2
B
17
A
32
A
47
B
62
C

[AIEEE-2008]

EXERCISE -5

ANSWER KEY
3
A
18
A
33
A
48
B
63
B

4
B
19
C
34
B
49
B

5
A
20
C
35
A
50
D

6
A
21
A
36
B
51
C

7
C
22
B
37
A
52
D

8
A
23
C
38
D
53
D

9
C
24
A
39
B
54
B

10
B
25
C
40
B
55
B

11
C
26
B
41
C
56
D

12
B
27
A
42
B
57
C

13
A
28
A
43
B
58
B

14
C
29
C
44
D
59
D

15
B
30
B
45
A
60
A

49

R E

-F O

U N D A T IO N

B Y

MANISH

MATHEMATICS

KUMAR
0
00.0

6
00.1

12,
00.2

18
00.3

24
00.4

Sin x0
30,
36
00.5 00.6

0000
0175
0349
0523
0698
0872
1045
1219
1392
1564
1736
1908
2079
2250
2419
2588
2756
2924
3090
3256
3420
3584
3746
3907
4067
4226
4384
4540
4695
4848
5000
5150
5299
5446
5592
5736
5878
6018
6157
6293
6428
6561
6691
6820
6947
7071

0017
0192
0366
0541
0715
0889
1063
1236
1409
1582
1754
1925
2096
2267
2436
2605
2773
2940
3107
3272
3437
3600
3762
3923
4083
4242
4399
4555
4710
4863
5015
5165
5314
5461
5606
5750
5892
6032
6170
6307
6441
6574
6704
6833
6959
7083

0035
0209
0384
0558
0732
0906
1080
1253
1426
1599
1771
1942
2113
2284
2453
2622
2790
2957
3123
3289
3453
3616
3778
3939
4099
4258
4415
4571
4726
4879
5030
5080
5329
5476
5621
5764
5906
6046
6184
6320
6455
6587
6717
6845
6972
7096

0052
0227
0401
0576
0750
0924
1097
1271
1444
1616
1788
1959
2300
2130
2470
2639
2807
2974
3140
3305
3469
3633
3795
3955
4115
4274
4431
4586
4741
4894
5045
5195
5344
5490
5635
5779
5920
6060
6198
6334
6468
6600
6730
6858
6984
7108

0070
0244
0419
0593
0767
0941
1115
1288
1461
1633
1805
1977
2147
2317
2487
2656
2823
2990
3156
3322
3486
3649
3811
3971
4131
4289
4436
4602
4756
4909
5060
5210
5358
5505
5650
5793
5934
6074
6211
6347
6481
6613
6743
6871
6997
7120

0087
0262
0436
0610
0785
0958
1132
1305
1478
1650
1822
1994
2164
2334
2504
2672
2840
3007
3173
3338
3502
3665
3827
3987
4147
4305
4462
4617
4772
4924
5075
5225
5373
5519
5664
5807
5948
6088
6225
6361
6494
6626
6756
6884
7009
7133

Degr
ee

.0
.1
.2
.3
.4
.5
.6
.7
.8
.9
.10
.11
.12
.13
.14
.15
.16
.17
.18
.19
.20
.21
.22
.23
.24
.25
.26
.27
.28
.29
.30
.31
.32
.33
.34
.35
.36
.37
.38
.39
.40
.41
.42
.43
.44
.45

0105
0279
0454
0628
0802
0976
1149
1323
1495
1668
1840
2011
2181
2351
2521
2689
2857
3024
3190
3355
3518
3681
3843
4003
4163
4321
4478
4433
4787
4939
5090
5240
5388
5534
5678
5821
5962
6101
6239
6374
6508
6639
6769
6896
7022
7145

42
00.7

48,
00.8

54
00.9

0122
0297
0471
1645
1819
1993
1167
1340
1513
1685
1857
2028
2198
2368
2538
2706
2874
3040
3206
3371
3535
3697
3859
4019
4179
4337
4493
4648
4802
4955
5105
5255
5402
5548
5693
5835
5976
6115
6252
6388
6521
6652
6782
6909
7034
7157

0140
0314
0488
0663
2837
1011
1184
1357
1530
1702
1874
2045
2215
2385
2554
2723
2890
3057
3223
3387
3551
3714
3875
4035
4195
4352
4509
4664
4818
4970
5120
5270
5417
5563
5707
5850
5850
5990
6129
6266
6401
6534
6665
6794
6921
7046

0157
0332
0506
0680
0854
1028
1201
1374
1547
1719
1891
2062
2233
2402
2571
2740
2907
3074
3239
3404
3567
3730
3891
4051
4210
4368
4524
4679
4833
4985
5135
5284
5432
5577
5721
5864
6004
6143
6280
6414
6547
6678
6807
6934
7059
7181

MEAN DEFFERENCES
1
3
3
3
3
3
3
3
3
3
3
3
3
3
3
3
3
3
3
3
3
3
3
3
3
3
3
3
3
3
3
2
2
2
2
2
2
2
2
2
2
2
2
2
2
2
2

2
6
6
6
6
6
6
6
6
6
6
6
6
6
6
6
6
6
6
6
5
5
5
5
5
5
5
5
5
5
5
5
5
5
5
5
5
5
5
5
4
4
4
4
4
4
4

3
9
9
9
9
9
9
9
9
9
9
9
9
9
8
8
8
8
8
8
8
8
8
8
8
8
8
8
8
8
8
8
7
7
7
7
7
7
7
7
7
7
7
6
6
6
6

4
12
12
12
12
12
12
12
12
12
12
11
11
11
11
11
11
11
11
11
11
11
11
11
11
11
11
10
10
10
10
10
10
10
10
10
9
9
9
9
9
9
9
9
8
8
8

5
15
15
15
15
15
14
14
14
14
14
14
14
14
14
14
14
14
14
14
14
14
14
14
14
13
13
13
13
13
13
13
12
12
12
12
12
12
12
12
11
11
11
11
11
10
10

50

R E

-F O

U N D A T IO N

B Y

MANISH

MATHEMATICS

KUMAR
0
00.0

6
00.1

12,
00.2

18
00.3

24
00.4

Sin x0
30,
36
00.5 00.6

7193
7314
7431
7547
7660
7771
7880
7986
8090
8192
8290
8387
8480
8572
8660
8746
8829
8910
8988
9063
9135
9205
9272
9336
9397
9455
9511
9563
9613
9659
9403
9744
9781
9816
9848
9877
9903
9925
9945
9962
9976
9986
9994
9998

7206
7325
7443
7559
7672
7782
7891
7997
8100
8202
8300
8396
8490
8581
8669
8755
8838
8318
8996
9070
9193
9212
9278
9342
9403
9461
9516
9568
9617
9664
9707
9748
9785
9820
9851
9880
9905
9928
9947
9963
9977
9987
9995
9999

7218
7337
7455
7570
7683
7793
7902
8007
8111
8211
8310
8406
8499
8590
8678
8763
8846
8926
9003
9078
9150
9219
9285
9348
9409
9466
9521
9573
9622
9668
9711
9751
9789
9823
9854
9882
9907
9930
9949
9965
9978
9988
9995
9999

7230
7349
7466
7581
7694
7804
7912
8018
8121
8221
8320
8415
8508
8599
8686
8771
8854
8934
9011
9085
9157
9225
9291
9354
9415
9472
9527
9578
9627
9673
9715
9755
9792
9826
9857
9885
9910
9932
9951
9966
9979
9989
9996
9999

7242
7361
7478
7593
7705
7815
7923
8028
8131
8231
8329
8425
8517
8607
8695
8780
8862
8942
9018
9092
9194
9232
9298
9361
9421
9478
9532
9583
9632
9677
9720
9759
9796
9829
9860
9888
9912
9934
9952
9968
9980
9990
9996
9999

7256
7373
7490
7604
7716
7826
7934
8039
8141
8241
8339
8434
8526
8616
8704
8788
8870
8949
9026
9100
9171
9239
9304
9367
9426
9483
9537
9588
9636
9681
9724
9763
9799
9833
9863
9890
9914
9936
9954
9969
9981
9990
9997
1.000

Degre
e

.46
.47
.48
.49
.50
.51
.52
.53
.54
.55
.56
.57
.58
.59
.60
.61
.62
.63
.64
.65
.66
.67
.68
.69
.70
.71
.72
.73
.74
.75
.76
.77
.78
.79
.80
.81
.82
.83
.84
.85
.86
.87
.88
.89

7266
7385
7501
7615
7727
7837
7944
8049
8151
8251
8348
8443
8536
8625
8712
8796
8878
8957
9033
9107
9178
9245
9311
9373
9432
9489
9542
9593
9641
9686
9728
9767
9803
9836
9866
9893
9917
9938
9956
9971
9982
9991
9997
1.000

42
00.7

48,
00.8

54
00.9

7278
7396
7513
7627
7738
7848
7955
8059
8161
8261
8358
8453
8545
8634
8721
8805
8886
8965
9041
9114
9184
9252
9317
9379
9438
9494
9548
9598
9646
9690
9732
9770
9806
9839
9869
9895
9919
9940
9957
9972
9983
9992
9997
1.000

7290
7408
7524
7638
7749
7859
7965
8070
8171
8271
8368
8462
8554
8643
8729
8813
8894
8973
9048
9121
9191
9259
9323
9385
9444
9500
9553
9603
9650
9694
9736
9774
9810
9842
9871
9898
9921
9942
9959
9973
9984
9993
9998
1.000

7302
7420
7536
7649
7760
7869
7976
8080
8181
8281
8377
8471
8563
8652
8738
8821
8902
8980
9056
9128
9198
9265
9330
9391
9449
9505
9558
9608
9655
9699
9740
9778
9813
9845
9874
9900
9923
9943
9960
9974
9985
9993
9998
1.000

MEAN DEFFERENCES
1
2
2
2
2
2
2
2
2
2
2
2
2
2
1
1
1
1
1
1
1
1
1
1
1
1
1
1
1
1
1
1
1
1
1
0
0
0
0
0
0
0
0
0
0

2
4
4
4
4
4
4
4
3
3
3
3
3
3
3
3
3
3
3
3
2
2
2
2
2
2
2
2
2
2
1
1
1
1
1
1
1
1
1
1
0
0
0
0
0

3
6
6
6
6
6
5
5
5
5
5
5
5
5
4
4
4
4
4
4
4
3
3
3
3
3
3
3
2
2
2
2
2
2
2
1
1
1
1
1
1
1
0
0
0

4
8
8
8
8
7
7
7
7
7
7
6
6
6
6
6
6
5
5
5
5
5
4
4
4
4
4
3
3
3
3
3
3
2
2
2
2
2
1
1
1
1
1
0
0

5
10
10
10
9
9
9
9
9
8
8
8
8
8
7
7
7
7
6
6
6
6
6
5
5
5
5
4
4
4
4
3
3
3
3
2
2
2
2
2
1
1
1
0
0

51

R E

-F O

U N D A T IO N

B Y

MANISH

MATHEMATICS

KUMAR

Degree

0
00.0

.0
.1
.2
.3
.4
.5
.6
.7
.8
.9
.10
.11
.12
.13
.14
.15
.16
.17
.18
.19
.20
.21
.22
.23
.24
.25
.26
.27
.28
.29
.30
.31
.32
.33
.34
.35
.36
.37
.38
.39
.40
.41
.42
.43
.44
.45

1.000
.9998
.9994
.9986
.9976
.9962
.9945
.9925
.9903
.9877
.9848
.9816
.9781
.9744
.9703
.9659
.9613
.9563
.9511
.9455
.9397
.9336
.9272
.9205
.9135
.9063
.8988
.8910
.8829
.8746
.8660
.8575
.8480
.8387
.8290
.8192
.8090
.7986
.7880
.7771
.7660
.7547
.7431
.7314
.7193
.7071

6
00.1
1.000
9998
9993
9985
9974
9960
9943
9923
9900
9874
9845
9813
9778
9740
9699
9655
9608
9558
9505
9449
9391
9330
9265
9198
9128
9056
8980
8902
8821
8738
8652
8563
8471
8377
8281
8181
8080
7976
7859
7760
7649
7536
7420
7302
7181
7059

12,
00.2
1.000
9998
9993
9984
9973
9959
9942
9921
9898
9871
9842
9810
9774
9736
9694
9650
9603
9553
9500
9444
9385
9323
5259
9191
9121
9048
8973
8894
8813
8729
8643
8554
8462
8368
8271
8171
8070
7965
7869
7749
7638
7424
7408
7290
7169
7046

18
00.3
1.000
9997
9992
9983
9972
9957
9940
9919
9895
9869
9839
9806
9770
9732
9690
9646
9598
9548
9494
9438
9379
9317
9252
9184
9114
9041
8965
8886
8805
8721
8634
8545
8453
8358
8261
8161
8059
7955
7848
7738
7627
7513
7396
7278
7157
7034

24
00.4
1.000
9997
9991
9982
9971
9956
9938
9917
9893
9866
9836
9805
9767
9728
9686
9641
9593
9542
9489
9432
9373
9311
9245
9178
9107
9033
8957
8878
8796
8712
8625
8536
8443
8343
8251
8151
8049
7944
7837
7727
7615
7501
7385
7266
7145
7022

30,
00.5
1.000
9997
9990
9981
9969
9954
9936
9914
9890
9863
9833
9799
9763
9724
9681
9636
9588
9537
9583
9426
9367
9304
9239
9171
9100
9026
8949
8870
8788
8704
8616
8526
8434
8339
8241
8141
8039
7934
7826
7716
7604
7490
7373
7254
7133
7009

36
00.6
9999
9996
9990
9980
9968
9952
9934
9912
9888
9860
9829
9796
9759
9720
9677
9632
9583
9532
9478
9421
9361
9298
9232
9164
9092
9018
8942
8862
8780
8695
8607
8517
8425
8329
8231
8131
8028
7923
7815
7705
7593
7478
7361
7242
7120
6997

42
00.7
9999
9996
9989
9966
9979
9951
9932
9910
9885
9857
9826
9792
9755
9715
9673
9627
9578
9527
9472
9415
9354
9291
9225
9157
9085
9011
8934
8854
8771
8686
8599
8508
8415
8320
8221
8121
8018
7912
7804
7694
7581
7466
7349
7230
7108
6984

48,
00.8
9999
9995
9988
9978
9965
9949
9930
9907
9882
9854
9823
9789
9751
9711
9668
9622
9573
9521
9466
9409
9348
9285
9219
9150
9078
9003
8926
8846
8763
8678
8590
8499
8406
8310
8211
8111
8007
7902
7793
7683
7570
7455
7337
7218
7096
6972

54
00.9
9999
9995
9987
9977
9963
9947
9928
9905
9880
9851
9820
9785
9748
9707
9664
9617
9568
9516
9461
9403
9342
9278
9212
9143
9070
8996
8918
8838
8755
8669
8581
8490
8396
8300
8202
8100
7997
7891
7782
7672
7559
7443
7325
7206
7083
6959

MEAN EFFERENCES
1
0
0
0
0
0
0
0
0
0
0
1
1
1
1
1
1
1
1
1
1
1
1
1
1
1
1
1
1
1
1
1
2
2
2
2
2
2
2
2
2
2
2
2
2
2
2

2
0
0
0
0
0
1
1
1
1
1
1
1
1
1
1
2
2
2
2
2
2
2
2
2
2
3
3
3
3
3
3
3
3
3
3
3
3
4
4
4
4
4
4
4
4
4

3
0
0
0
1
1
1
1
1
1
1
2
2
2
2
2
2
2
3
3
3
3
3
3
3
3
4
4
4
4
4
4
5
5
5
5
5
5
5
5
6
6
6
6
6
6
6

4
0
0
1
1
1
1
1
2
2
2
2
2
3
3
3
3
3
3
4
4
4
4
4
5
5
5
5
5
6
6
6
6
6
6
7
7
7
7
7
7
8
8
8
8
8
8

52

5
0
0
1
1
1
2
2
2
2
2
3
3
3
3
4
4
4
4
5
5
5
5
6
6
6
6
6
7
7
7
7
8
8
8
8
8
9
9
9
9
9
10
10
10
10
10

R E

-F O

U N D A T IO N

B Y

MANISH

MATHEMATICS

KUMAR

Degree

0
00.0

.46
.47
.48
.49
.50
.51
.52
.53
.54
.55
.56
.57
.58
.59
.60
.61
.62
.63
.64
.65
.66
.67
.68
.69
.70
.71
.72
.73
.74
.75
.76
.77
.78
.79
.80
.81
.82
.83
.84
.85
.86
.87
.88
.89

.6947
.6820
.6691
.6428
.6428
.9293
.6157
.6018
.5878
.5736
.5592
.5446
.5299
.5150
.5000
.4848
.4695
.4540
.4384
.4226
.4067
.3907
.3746
.3584
.3420
.3256
.3090
.2924
.2756
.2588
.2419
.2250
.2079
.1908
.1736
.1564
.1392
.1219
.1045
.0872
.0698
.0523
.0349
.0175

6
00.1
6934
6807
6678
6547
6414
6280
6124
6004
5864
5721
5577
5432
5284
5135
4985
4833
4679
4524
4368
4210
4051
3891
3730
3567
3404
3239
3074
2907
2740
2571
2402
2233
2062
1891
1719
1547
1374
1201
1028
0854
0680
0506
0332
0157

12,
00.2
6921
6794
6665
6534
6401
6266
6129
5990
5850
5707
5563
5417
5270
5120
4970
4818
4664
4509
4352
4195
4035
3875
3714
3551
3387
3223
3057
2890
2723
2554
2385
2215
2045
1874
1702
1530
1357
1184
1011
0837
0663
0488
0314
0140

18
00.3
6909
6782
6652
6521
3688
6252
6115
5976
5835
5693
5548
5402
5255
5105
4955
4802
4648
4493
4337
4179
4019
3859
3697
3535
3371
3206
3040
2874
2706
2538
2368
2198
2028
1857
1685
1513
1340
1167
0993
0819
0645
0471
0297
0122

24
00.4
6896
6769
6639
6508
6374
6239
6101
5962
5821
5678
5534
5388
5240
5090
4939
4787
4633
4478
4321
4163
4003
3843
3681
3518
3335
3190
3024
2857
2689
2521
2351
2181
2011
1840
1668
1495
1323
1149
0976
0802
0623
0454
0279
0105

30,
00.5
6884
6756
6626
6494
6361
6225
6088
5948
5807
5664
5519
5373
5225
5075
4924
4772
4617
4462
4305
4147
3987
3887
3665
3502
3338
3173
3007
2840
2672
2504
2334
2164
1994
1822
1650
1478
1305
1132
0958
0785
0610
0436
0262
0087

36
00.6
6871
6743
6613
6481
6347
6211
6074
5934
5793
5650
5505
5358
5210
5060
4909
4756
4602
4446
4289
4131
3971
3811
3649
3486
3322
3156
2990
2823
2656
2487
2317
2147
1977
1805
1633
1461
1288
1115
0941
0767
0593
0419
0244
0070

42
00.7
6858
6730
6600
6468
6334
6198
6060
5920
5779
5635
5490
5344
5135
5045
4894
4741
4586
4431
4274
4115
3955
3795
3633
3469
3305
3140
2974
2807
2639
2470
2300
2130
1959
1788
1616
1444
1271
1097
0924
0750
0576
0401
0227
0052

48,
00.8
6845
6717
6587
6455
6320
6184
6064
5906
5764
5621
5476
5329
5180
5030
4879
4726
4571
4415
4258
4099
3939
3778
3616
3453
3289
3123
2957
2790
2622
2453
2284
2113
1942
1771
1599
1426
1253
1080
0906
0732
0558
0384
0209
0035

54
00.9
6833
6704
6574
6441
6307
6170
6032
5892
5750
5606
5461
5314
5165
5015
4863
4710
4555
4399
4242
4083
3923
3762
3600
3437
3272
3107
2940
2773
2605
2436
2267
2096
1925
1754
1582
1409
1236
1063
0889
0715
0541
0366
0192
0017

MEAN EFFERENCES
1
2
2
2
2
2
2
2
2
2
2
2
2
2
3
3
3
3
3
3
3
3
3
3
3
3
3
3
3
3
3
3
3
3
3
3
3
3
3
3
3
3
3
3
3

2
4
4
4
4
4
5
5
5
5
5
5
5
5
5
5
5
5
5
5
5
5
5
5
5
5
6
6
6
6
6
6
6
6
6
6
6
6
6
6
6
6
6
6
6

3
6
6
7
7
7
7
7
7
7
7
7
7
7
8
8
8
8
8
8
8
8
8
8
8
8
8
8
8
8
8
8
9
9
9
9
9
9
9
9
9
9
9
9
9

4
8
9
9
9
9
9
9
9
9
10
10
10
10
10
10
10
10
10
11
11
11
11
11
11
11
11
11
11
11
11
11
11
11
11
12
12
12
12
12
12
12
12
12
12

53

5
11
11
11
11
11
11
12
12
12
12
12
12
12
12
13
13
13
13
13
13
14
14
14
14
14
14
14
14
14
14
14
14
14
14
14
14
14
14
14
14
15
15
15
15

R E

-F O

U N D A T IO N

B Y

MANISH

MATHEMATICS

KUMAR
0
00.0

6
00.1

12,
00.2

18
00.3

24
00.4

tan x0
30,
36
00.5 00.6

0000
0175
0349
0524
0699
0875
1051
1228
1405
1584
1763
1944
2126
2309
2493
2679
2867
3057
3249
3443
3640
3839
4040
4245
4452
4663
4877
5095
5317
5543
5774
6009
6249
6494
6745
7002
7265
7536
7513
8098
8391
8693
9004
9325
9657
1.000

0017
0192
0367
0542
0717
0892
1069
1246
1423
1602
1781
1962
2144
2327
2512
2698
2886
3076
3269
3463
3659
3859
4061
4265
4473
4684
4809
5117
5340
5566
5797
6032
6273
6519
6771
7028
7292
7563
7841
8127
8421
8724
9036
9358
9691
0035

0035
0209
0384
0559
0734
0910
1086
1263
1441
1620
1799
1980
2162
2345
2530
2717
1905
3096
3288
3482
3679
3869
4081
4286
4494
4706
4921
5139
5362
5589
5820
6056
6297
6544
6796
7054
7319
7590
7869
8156
8451
8754
9067
9391
9725
0070

0052
0227
0402
0577
0752
0928
1104
1281
1459
1638
1817
1998
2180
2364
2549
2736
2924
3115
3307
3502
3699
3899
4101
4307
4515
4727
4942
5161
5384
5612
5844
6080
6322
6569
6822
7080
7346
7618
7898
8185
8481
8785
9099
9424
9759
0105

0070
0244
0419
0594
0769
0945
1122
1299
1477
1635
1835
2016
2199
2382
2568
2754
2943
3134
3327
3522
3719
3919
4122
4327
4536
4748
4962
5184
5407
5635
5867
6104
6346
6594
6874
7107
7373
7646
7926
8214
8511
8816
9131
9457
9793
0141

0087
0262
0437
0612
0787
0963
1139
1317
1495
1673
1853
2035
2217
2401
2586
2773
2962
3253
3541
3739
4142
4348
4557
4770
4986
5206
5430
5658
5890
6128
6371
6619
6873
7133
7400
7673
7400
7673
7954
8243
8541
8847
9163
9490
9827
0176

Degr
ee

.0
.1
.2
.3
.4
.5
.6
.7
.8
.9
.10
.11
.12
.13
.14
.15
.16
.17
.18
.19
.20
.21
.22
.23
.24
.25
.26
.27
.28
.29
.30
.31
.32
.33
.34
.35
.36
.37
.38
.39
.40
.41
.42
.43
.44
.45

0105
0279
0454
0629
0805
0981
1157
1334
1512
1691
1871
2053
2235
2419
2605
2792
2981
3172
3365
3561
3759
3959
4163
4369
4578
4791
5008
5228
5452
5681
5914
6152
6395
6644
6899
7159
7427
7701
7983
8273
8571
8878
9195
9523
9861
0212

42
00.7

48,
54
00.8 00.9

0122
0297
0472
0647
0822
0998
1175
1352
1530
1709
1890
2071
2254
2438
2623
2811
3000
3191
3385
3581
3779
3979
4183
4390
4599
4813
5029
5250
5475
5704
5938
6176
6420
6669
6924
7186
7454
7729
8012
8302
8601
8910
9228
9556
9896
0247

0140
0314
0489
0664
0840
1016
1192
1370
1548
1727
1908
2089
2272
2456
6242
2830
3019
3211
3404
3600
3799
4000
4204
4411
4621
4834
5051
5272
5498
5727
5961
6200
6445
6694
6950
7212
7481
7757
8040
8332
8632
8941
6260
9590
9930
0283

0157
0332
0507
0682
0857
1033
1210
1388
1566
1745
1926
2107
2290
2475
2661
2849
3038
3230
3424
3620
3819
4020
4224
4431
4642
4856
5073
5295
5520
5750
5985
6224
6469
6720
6976
7239
7508
7785
8069
8361
8662
8972
9293
9623
9965
0319

MEAN DEFFERENCES
1
3
3
3
3
3
3
3
3
3
3
3
3
3
3
3
3
3
3
3
3
3
3
3
3
4
4
4
4
4
4
4
4
4
4
4
4
5
5
5
5
5
5
5
6
6
6

2
6
6
6
6
6
6
6
6
6
6
6
6
6
6
6
6
6
6
6
7
7
7
7
7
7
7
7
7
8
8
8
8
8
8
9
9
9
9
9
10
10
10
11
11
11
12

3
9
9
9
9
9
9
9
9
9
9
9
9
9
9
9
9
9
10
10
10
10
10
10
10
11
11
11
11
11
12
12
12
12
13
13
13
14
14
14
15
15
16
16
17
17
18

4
12
12
12
12
12
12
12
12
12
12
12
12
12
12
12
13
13
13
13
13
13
13
14
14
14
14
15
15
15
15
16
16
16
17
17
18
18
18
19
20
20
21
21
22
23
24

5
15
15
15
15
15
15
15
15
15
15
15
15
15
15
16
16
16
16
16
16
17
17
17
17
18
18
18
18
19
19
20
20
20
21
21
22
23
23
24
24
25
26
27
28
29
30

54

R E

-F O

U N D A T IO N

B Y

MANISH

MATHEMATICS

Degree

KUMAR

0
00.0

6
00.1

12,
00.2

18
00.3

24
00.4

30,
00.5

.46
.47
.48
.49
.50
.51
.52
.53
.54
.55
.56
.57
.58
.59
.60
.61
.62
.63
.64
.65
.66
.67
.68
.69
.70
.71
.72
.73
.74
.75
.76
.77
.78
.79
.80
.81
.82
.83
.84
.85
.86
.87
.88
.89

1.0355
1.0724
1.1106
1.1504
1.1918
1.2349
1.2799
1.3270
1.3764
1.4281
1.4826
1.5399
1.6003
1.6643
1.7321
1.8040
1.8807
1.9626
2.0503
2.1445
2.2460
2.3559
2.4751
2.6051
2.7475
2.9042
3.0777
.02709
3.4874
3.7321
4.0108
4.3315
4.7046
5.1446
5.6713
6.3138
7.1154
8.1443
9.5144
11.43
14.30
19.08
28.64
57.29

0392
0761
1145
1544
1960
2393
2846
3319
3814
4335
4882
5458
6066
6709
7391
8115
8887
9711
0594
1543
2566
3673
4876
6187
7625
9208
0961
2914
5105
7583
0408
3662
7453
1923
7297
3859
2066
2636
9.677
11.66
14.67
19.74
30.14
63.66

0428
0299
1184
1585
2002
2437
2892
3367
3865
4388
4938
4938
5517
6128
6775
7461
8190
8967
9797
0686
1642
2673
3789
5002
6325
7776
9375
1146
3122
5339
7848
0713
4015
7867
2422
7894
4596
3002
9.845
11.91
15.06
20.45
31.82
71.62

0464
0837
1224
1626
2045
2484
2938
3416
3916
4442
4994
5577
6191
6842
7532
8265
9047
9883
0778
1742
2781
3906
5129
6464
7929
9544
1334
3332
5576
8118
1022
4374
8288
2924
8502
5350
3962
5126
10.02
12.16
15.46
21.20
33.69
81.85

0501
0875
1263
1667
2088
2527
2985
3465
3968
4496
5051
5637
6255
6909
7603
8341
9128
9970
0872
1842
2889
4023
5257
6605
8083
9714
1524
3544
5816
8391
1335
4737
8716
3435
9124
6122
4947
6427
10.20
12.43
15.89
22.02
35.80
95.49

0838
0913
1303
1708
2131
2572
3032
3514
4019
4550
5108
5697
6319
6977
7675
8418
9210
2.0057
0965
1943
2998
4142
5386
6746
8239
9887
1716
3759
6059
8667
1653
5107
9152
3955
9758
6912
5958
7769
10.39
12.71
16.35
22.90
38.19
114.6

tan x0
36
42
00.6
00.7
0575
0951
1343
1750
2174
2617
3079
3564
4071
4605
5166
5757
6383
7045
7747
8405
9262
2.0145
1060
2045
3109
4262
5517
6889
8397
3.0061
1910
3977
6305
8947
1976
5483
9594
4486
6.0405
7720
6996
9152
10.58
13.00
16.83
23.86
40.62
143.2

0612
0990
1383
1792
2218
2662
3127
3613
4124
4659
5224
5818
6447
7113
7820
8572
9375
2.0233
1155
2148
3220
4383
5649
7034
8556
3.0237
2106
4197
6554
9232
2303
5864
5.0045
5026
6.1066
8548
8062
9.0579
10.78
13.30
17.34
24.90
44.07
191.0

48,
00.8
0649
1028
1423
1833
2261
2708
3175
3663
4176
4715
5282
5880
6512
7182
7893
8650
9458
2.0323
1155
2148
3220
4383
5649
7034
8556
3.0237
2106
4197
6554
9232
2303
5864
5.0045
5026
6.1066
8548
8062
9.0579
10.78
13.30
17.34
24.90
44.07
191.0

54
00.9
0686
1067
1463
1875
2305
2753
3222
3713
4229
4770
5340
5941
6577
7251
7966
8728
9542
2.0413
1348
2355
3445
4627
5916
7326
8878
3.0595
2506
4646
7062
9812
2972
6646
5.0970
6140
6.2432
7.0264
8.0285
9.3572
11.20
13.95
18.46
27.27
52.08
573.0

MEAN EFFERENCES
1
6
6
7
7
7
8
8
8
9
9
10
10
11
11
12
13
14
15
16
17
18
20
22
24
26
29
32
36
41
46
53

2
12
13
13
14
14
15
16
16
17
18
19
20
21
23
24
26
27
29
31
34
37
40
43
47
52
58
64
72
81
93
107

3
18
19
20
21
22
23
24
25
26
27
29
30
32
34
36
38
41
44
47
51
55
60
65
71
78
87
96
108
122
139
160

4
25
25
27
28
29
30
31
33
34
36
38
40
43
45
48
51
55
58
63
68
73
79
87
95
104
116
129
144
163
186
213

55

5
31
32
33
34
36
38
39
41
43
45
48
50
53
56
60
64
68
73
78
85
92
99
108
119
133
145
161
180
204
232
267

You might also like